insight test-13 xaam.in

66
TEST - 13 User Name : chandan paswan Total Marks : 200 Mark Scored : 22 (C) Insights Active Learning. | All rights reserved. www.insightsias.com 1 1 The key difference(s) between Reserved Forests and Protected Forests is/are that All activities are permitted unless restricted in protected forests, whereas in reserved 1. forests all activities are restricted unless permitted Reserved forests are declared by Union government whereas protected forests are 2. declared by state governments. Reserved forests are owned by private individuals, protected forests are not. 3. Select the correct answer using the codes below. 1 only A. 1 and 3 only B. 2 and 3 only C. 1, 2 and 3 D. User Answer : A Correct Answer : A Answer Justification : Justification: Rights to all activities like hunting, grazing, etc. in reserved forests are banned unless specific orders are issued otherwise. In protected areas, rights to activities like hunting and grazing are sometimes given to communities living on the fringes of the forest, who sustain their livelihood partially or wholly from forest resources or products. So, 1 is correct. Both can be declared by the state government. So, 2 is wrong. Unclassed forests (not reserved or protected) are other forests and wastelands belonging to both government and private individuals and communities. So, 3 is also wrong. Q Source: 12th NCERT: Geography: India, People and Economy: Glossary 2 The Veda Samaj worked to Abolish caste distinctions 1. Rationalize superstitious Hindu practices 2. Open Universities that promote the study of Eastern mysticism 3. Promote the idea of God Trinity 4. Select the correct answer using the codes below. 1 only A. 1 and 2 only B. 2 and 4 only C. 1, 2, 3 and 4 D. User Answer : A INSIGHTS IAS

Upload: vinayelectrical

Post on 15-Jul-2016

44 views

Category:

Documents


5 download

DESCRIPTION

UPSC test 2016

TRANSCRIPT

Page 1: Insight Test-13 Xaam.in

TEST - 13

User Name : chandan paswan

Total Marks : 200

Mark Scored : 22

(C) Insights Active Learning. | All rights reserved. www.insightsias.com 1

1 The key difference(s) between Reserved Forests and Protected Forests is/are thatAll activities are permitted unless restricted in protected forests, whereas in reserved1.forests all activities are restricted unless permittedReserved forests are declared by Union government whereas protected forests are2.declared by state governments.Reserved forests are owned by private individuals, protected forests are not.3.Select the correct answer using the codes below.

1 onlyA.1 and 3 onlyB.2 and 3 onlyC.1, 2 and 3D.

User Answer : ACorrect Answer : AAnswer Justification :

Justification: Rights to all activities like hunting, grazing, etc. in reserved forestsare banned unless specific orders are issued otherwise. In protected areas, rights toactivities like hunting and grazing are sometimes given to communities living on thefringes of the forest, who sustain their livelihood partially or wholly from forestresources or products. So, 1 is correct.

Both can be declared by the state government. So, 2 is wrong.

Unclassed forests (not reserved or protected) are other forests and wastelandsbelonging to both government and private individuals and communities. So, 3 is alsowrong.

Q Source: 12th NCERT: Geography: India, People and Economy: Glossary

2 The Veda Samaj worked toAbolish caste distinctions1.Rationalize superstitious Hindu practices2.Open Universities that promote the study of Eastern mysticism3.Promote the idea of God Trinity4.Select the correct answer using the codes below.

1 onlyA.1 and 2 onlyB.2 and 4 onlyC.1, 2, 3 and 4D.

User Answer : A

INSIG

HTS IAS

Page 2: Insight Test-13 Xaam.in

TEST - 13

User Name : chandan paswan

Total Marks 200

Mark Scored 22

(C) Insights Active Learning. | All rights reserved. www.insightsias.com 2

Correct Answer : AAnswer Justification :

Justification & Learning: It was established in Madras (Chennai) in 1864.

It was inspired by the Brahmo Samaj and worked to abolish caste distinctions andpromote widow remarriage and women's education. So, 1 is correct.

Its members believed in one God. They condemned the superstitions and rituals oforthodox Hinduism. So, 2 and 4 are incorrect.

Q Source: 8th NCERT: History - Part 2

3 Water absorption through roots can be increased by keeping the plantsCovered with a plastic bagA.In LED lightB.Under the fanC.In dark shadeD.

User Answer :Correct Answer : CAnswer Justification :

Justification & Learning: Transpiration is the process by which moisture is carriedthrough plants from roots to small pores on the underside of leaves, where it changesto vapor and is released to the atmosphere.

Increased movement of the air around a plant will result in a higher transpirationrate. This is somewhat related to the relative humidity of the air, in that as watertranspires from a leaf, the water saturates the air surrounding the leaf.

If there is no wind, the air around the leaf may not move very much, raising thehumidity of the air around the leaf.

Wind will move the air around, with the result that the more saturated air close tothe leaf is replaced by drier air. So, (c) is correct.

If a plant is kept in conditions (a), (b) or (d), transpiration rate would not increase.

Q Source: 6th Science NCERT

4 Consider the following statements.

INSIG

HTS IAS

Page 3: Insight Test-13 Xaam.in

TEST - 13

User Name : chandan paswan

Total Marks : 200

Mark Scored : 22

(C) Insights Active Learning. | All rights reserved. www.insightsias.com 3

The nearest major town is Joshimath.1.There is no settlement in the national park.2.It is encompassed in the Nanda Devi Biosphere Reserve.3.It is in the UNESCO World Network of Biosphere Reserves.4.It is known for its meadows of endemic alpine flowers.5.The above refer to?

Dudhwa National ParkA.Valley of Flowers National ParkB.Gangotri National ParkC.Indravati National ParkD.

User Answer :Correct Answer : BAnswer Justification :

Justification: Dudhwa is in U.P, whereas Joshimath is in Uttarakhand. So, (a) iseliminated.

While Gangotri park is in Uttarakhand, Harsil is the nearest town and it is not in theUNESCO network. So, (c) is also eliminated.

Indravati is in MP, so (d) is also wrong.

Learning: Located in West Himalayas, it is home to rare and endangered animals,including the Asiatic black bear, snow leopard, musk deer, brown bear, red fox andblue sheep.

Flowers like orchids, poppies, primulas, marigold, daisies and anemones carpet theground at the national park.

Q Source: Test 13 Syllabus - National Parks

5 Consider the following statements about the United Nations High Commissioner forRefugees (UNHCR).

It is a member of the United Nations Development Group (UNDP).1.It can protect and support refugees even at the request of a national government.2.Which of the above is/are correct?

1 onlyA.2 onlyB.Both 1 and 2C.NoneD.

User Answer :

INSIG

HTS IAS

Page 4: Insight Test-13 Xaam.in

TEST - 13

User Name : chandan paswan

Total Marks 200

Mark Scored 22

(C) Insights Active Learning. | All rights reserved. www.insightsias.com 4

Correct Answer : CAnswer Justification :

Justification & Learning: It is a United Nations agency mandated to protect andsupport refugees.

It is a member of the United Nations Development Group. Initially it wasestablished to help people displaced by World War II. But later it became theprincipal agency that has helped displaced persons all over world.It is mandated to protect and support refugees at the request of a governmentor the UN itself and assists in their voluntary repatriation, local integration orresettlement to a third country.

Q Source: Frequently in news due to the Refugee crisis in Syria and Europe

6 Maintaining a Cash Reserve Ratio (CRR) is NOT mandatory for which of the followingbanks in India?

Banks with capital base above or below a certain limit as prescribed by RBI1.Banks that lend majority of their capital to farmer cooperatives or vulnerable2.sectionsBanks that have sponsored a rural bank to promote financial inclusion and3.developmentSelect the correct answer using the codes below.

1 and 2 onlyA.2 and 3 onlyB.1 onlyC.None of the aboveD.

User Answer : DCorrect Answer : DAnswer Justification :

Justification: Maintaining CRR is compulsory for all banks irrespective of theconditions mentioned in statements 1, 2 and 3.

CRR is the amount that the banks have to keep with the RBI. The banks do not earnany interest rate on this CRR, which the banks also called dead asset.

CRR has been reduced from as high as 23% to less than 4% in the past few decades.

Q Source: 12th NCERT: Macroeconomics

INSIG

HTS IAS

Page 5: Insight Test-13 Xaam.in

TEST - 13

User Name : chandan paswan

Total Marks : 200

Mark Scored : 22

(C) Insights Active Learning. | All rights reserved. www.insightsias.com 5

7 Prithvi-II developed by DRDO under India's prestigious Integrated Guided MissileDevelopment Program is a surface-to-surface medium range ballistic missile. What do youunderstand by ballistic missiles?

Most of the trajectory of these missiles is unpowered and governed by gravityA.and air resistanceMissiles that are aerodynamically guided in a powered flightB.Missiles that spend most of their flight out of the atmosphereC.Missiles that are launched at a steep trajectory and can caused heavy damageD.to its target

User Answer :Correct Answer : AAnswer Justification :

Learning: A ballistic missile is a missile (rocket) that follows a ballistic trajectorywith the objective of delivering one or more warheads to a predetermined target.

A ballistic missile is only guided during relatively brief periods of flight (there areunguided ballistic missiles as well, although these may well be considered rockets),and most of its trajectory is unpowered and governed by gravity and air resistance ifin the atmosphere.

This contrasts to a cruise missile, which is aerodynamically guided in poweredflight.

Long range intercontinental ballistic missiles (ICBM) are launched at a steep, sub-orbital flight trajectory and spend most of their flight out of the atmosphere. Shorterrange ballistic missiles stay within the Earth's atmosphere.

Q Source:http://www.thehindu.com/news/national/nuclear-capable-prithviii-missile-testfired/article6911901.ece

8 'State of the World's Forests' report is published byUnited Nations Environment Programme (UNEP)A.Food and Agriculture Organization (FAO)B.United Nations Framework Convention on Climate Change (UNFCCC)C.United Nations Development Programme (UND)D.

User Answer :Correct Answer : BAnswer Justification :

INSIG

HTS IAS

Page 6: Insight Test-13 Xaam.in

TEST - 13

User Name : chandan paswan

Total Marks 200

Mark Scored 22

(C) Insights Active Learning. | All rights reserved. www.insightsias.com 6

Learning: The State of the World's Forests reports on the status of forests, recentmajor policy and institutional developments and key issues concerning the forestsector.

Some of its observations in 2014 report were:

Forest products make a significant contribution to the shelter of at least 1.3billion people, or 18 percent of the world's population.A major contribution of forests to food security and health is the provision ofwoodfuel to cook and sterilize water.The formal forest sector employs some 13.2 million people across the worldand at least another 41 million are employed in the informal sector.Recognition of the value of forest services, such as erosion protection andpollination, is essential to sound decision-making.

Q Source: Test-13 Syllabus - International Reports

9 Sovereign rights over Antarctic territory is exercised byUSA1.Chile2.Australia3.UK4.Select the correct answer using the codes below.

1 and 3 onlyA.2 and 3 onlyB.1, 2, 3 and 4C.None of the aboveD.

User Answer :Correct Answer : DAnswer Justification :

Justification: Antarctica is a global common and so no one nation exercisessovereignty over the territory.

It is governed under 1959 Antarctic Treaty, an international treaty, that barscountries from owning or exploiting its land. Since its signing by 45 nations, claimsof seven countries for territory in the region were suspended.

Q Source: Page 121: Chapter 8: 12th NCERT: Contemporary World Politics

INSIG

HTS IAS

Page 7: Insight Test-13 Xaam.in

TEST - 13

User Name : chandan paswan

Total Marks : 200

Mark Scored : 22

(C) Insights Active Learning. | All rights reserved. www.insightsias.com 7

10 What does Baudhayana Sulabautra relate to?Lengths of sides of a right-angled triangleA.Earth's movement around the SunB.Mathematical paradox of zeroC.Calculation of orbital period of planetsD.

User Answer :Correct Answer : AAnswer Justification :

Learning: The Baudhayana Sulbasutra is noted for containing several earlymathematical results, including an approximation of the square root of 2 and thestatement of a version of the Pythagorean theorem.

The Baudhayana sutras are a group of Vedic Sanskrit texts which cover dharma,daily ritual, mathematics, etc.

They belong to the Taittiriya branch of the Krishna Yajurveda school and are amongthe earliest texts of the sutra genre

Q Source: CSP 2008

11 Most of the explosions in mines happen due to the occurrence of?Methane and airA.Oxygen and acetyleneB.Hydrogen and oxygenC.Carbon dioxide and methaneD.

User Answer : ACorrect Answer : AAnswer Justification :

Justification & Learning: Methane explosions occur in mines when a build-up ofmethane gas, a by-product of coal, comes into contact with a heat source, and thereis not enough air to dilute the gas to levels below its explosion point.

This produces water, carbon dioxide and a lot of amount of heat.

The heat generated by this process raises the temperature of the air within the mine,which causes it to expand in volume. Since hot air cannot expand easilyunderground, pressure builds in the mine. If this pressure is high enough, it cancause the air ahead of the combustion zone to compress and cause a shock wave,

INSIG

HTS IAS

Page 8: Insight Test-13 Xaam.in

TEST - 13

User Name : chandan paswan

Total Marks 200

Mark Scored 22

(C) Insights Active Learning. | All rights reserved. www.insightsias.com 8

Q Source: CSP 2008

12 Which state of India shares boundary with maximum number of other Indian states andUnion Territories (UTs)?

AssamA.Madhya PradeshB.Uttar PradeshC.JharkhandD.

User Answer : CCorrect Answer : CAnswer Justification :

Justification: Uttar Pradesh shares boundary with Uttarakhand, Himachal Pradesh,Haryana, Rajasthan, Madhya Pradesh, Chhattisgarh, Jharkhand, Bihar as well asDelhi.

Among the Southern states Karnataka shares maximum number of boundaries withAP, Telangana, Goa, MH, Kerala and TN.

Assam shares boundary with WB, Meghalaya, Arunachal Pradesh, Nagaland,Manipur, Mizoram, Tripura.

Q Source: Previous year UPSC papers

13 Consider the following about Chipko movement.It was introduced primarily against exploitation of tenants.1.One of its major objectives was to nationalize all peasant forest land in India.2.It focussed on conservation of traditional values of Indian culture.3.Select the correct answer using the codes below.

1 and 2 onlyA.3 onlyB.2 and 3 onlyC.None of the aboveD.

User Answer : BCorrect Answer : DAnswer Justification :

Justification: It was primarily a forest conservation movement. It occurred at a timewhen there was hardly any environmental movement in the developing world.

INSIG

HTS IAS

Page 9: Insight Test-13 Xaam.in

TEST - 13

User Name : chandan paswan

Total Marks : 200

Mark Scored : 22

(C) Insights Active Learning. | All rights reserved. www.insightsias.com 9

The Garhwal Himalayas became its centre for a rising ecological awareness to showhow reckless deforestation had denuded much of the forest cover, resulting in thedevastating Alaknanda River floods of 1970.

The name of the movement comes from the word 'embrace', as the villagers huggedthe trees, and prevented the contractors' from felling them.

Q Source: Page 168: Chapter 9: 11th NCERT: Indian Economic Development

14 The Right to Information (RTI) Act, 2005 confers a right to the citizens to know howthe taxpayer's money is being spent by the Government. Consider the following about it.

Right to Information has been given the status of a fundamental right under Article1.19 of the Constitution.It applies to all States and Union Territories of India except Jammu & Kashmir.2.RTI includes NGOs which are substantially financed by the government.3.It does not cover the Judiciary.4.Information received in confidence from foreign Government is exempt from5.disclosure.Select the correct answer using the codes below.

2 and 3 onlyA.1, 3 and 4 onlyB.4 and 5 onlyC.1, 2, 3 and 5D.

User Answer :Correct Answer : DAnswer Justification :

Justification: It has been given the status of a fundamental right under Article 19(1)of the Constitution. Article 19 (1) under which every citizen has freedom of speechand expression and have the right to know how the government works, what roledoes it play, what are its functions and so on.

RTI Act extends to the whole of India (except the State of Jammu and Kashmir), allbodies, which come under Government notification including NGOs, which areowned, controlled or are substantially financed by the Government. So, 2 and 3 arecorrect.

It covers all constitutional authorities, including the executive, legislature andjudiciary; any institution or body established or constituted by an act of Parliamentor a state legislature. So, 4 is incorrect.

INSIG

HTS IAS

Page 10: Insight Test-13 Xaam.in

TEST - 13

User Name : chandan paswan

Total Marks 200

Mark Scored 22

(C) Insights Active Learning. | All rights reserved. www.insightsias.com 10

Read about the exclusions here

https://en.wikipedia.org/wiki/Right_to_Information_Act,_2005#Exclusions

Q Source: 10 years of RTI in 2015

15 Consider the following statements.Assertion (A): Unlike the Council of Ministers, the constitution does not mention1.the powers and functions of the cabinet.Reason (R): The cabinet does not find any mention in the constitution.2.In the context of the above, which of these is correct?

A is correct, and R is an appropriate explanation of A.A.A is correct, but R is not an appropriate explanation of A.B.A is correct, but R is incorrect.C.Both A and R are incorrect.D.

User Answer :Correct Answer : CAnswer Justification :

Justification: The word cabinet was inserted in the constitution in Article 352 in1978 by the 44th CA Act.

However, it only defines the cabinet, not its powers and functions. So, A is correct,and R is incorrect.

Union Cabinet is the supreme decision-making body in India. Only the PrimeMinister and ministers of the rank of "Cabinet Minister" are members of theCabinet.

Q Source: 11th NCERT: Indian constitution at Work

16 Consider the following about the Bahmani architecture.The Bahmanis were influenced by the style of the Delhi Sultanate.1.The military architecture of Europe influenced Bahmani designs.2.Some monuments of Bijapur showcase Bahmani architecture.3.Select the correct answer using the codes below.

1 and 2 onlyA.2 onlyB.2 and 3 onlyC.1, 2 and 3D.

INSIG

HTS IAS

Page 11: Insight Test-13 Xaam.in

TEST - 13

User Name : chandan paswan

Total Marks : 200

Mark Scored : 22

(C) Insights Active Learning. | All rights reserved. www.insightsias.com 11

User Answer :Correct Answer : DAnswer Justification :

Learning: As Muhammad bin Tughlaq transferred his capital to Daulatabad andthen back to Delhi, the artisans migrated to Bijapur and their two styles ofarchitecture, the Persian and that of Delhi got a new fusion style.

The Persian style of architecture affected them. The Persian architects as well astheir artisans planned all the architecture of various structures. However, the militaryarchitecture i, e, forts etc were modelled on medieval European style.

Persian influence is apparently visible in the Jama Masjid at Gulbarga, Chand Minarat Daulatabad and Gawan's Madrasa at Bidar.

Charminar of Hyderabad is a remarkable structure of Bahmani architecture.

Q Source: UPSC CSP 2011-13

17 Why loosening of soil is important for agriculture?It allows the roots to breathe easily.1.The loosened soil helps in the growth of earthworms and microbes present in the2.soil.Which of the above is/are correct?

1 onlyA.2 onlyB.Both 1 and 2C.NoneD.

User Answer : BCorrect Answer : CAnswer Justification :

Justification: Loose soil has more air trapped in it than tight soil. Roots get moreoxygen and loose space this way and grow properly. So, 1 is correct.

Tight soil prohibits the growth of microbes as well as their movement. Loosening ofsoil allows for microbe action which decomposes organic matter to form humus inthe soil. So, 2 is correct.

Learning: Since only a few centimetres of the top layer of soil supports plantgrowth, turning and loosening of soil brings the nutrient-rich soil to the top so that

INSIG

HTS IAS

Page 12: Insight Test-13 Xaam.in

TEST - 13

User Name : chandan paswan

Total Marks 200

Mark Scored 22

(C) Insights Active Learning. | All rights reserved. www.insightsias.com 12

plants can use these nutrients.

Q Source: 8th NCERT: Science

18 Consider the following statements.Pulses and vegetables can be grown during Kharif period1.Oilseeds cannot be grown during Rabi period.2.Which of the above is/are correct?

1 onlyA.2 onlyB.Both 1 and 2C.NoneD.

User Answer : CCorrect Answer : AAnswer Justification :

Justification: Arhar, Moong etc can be grown during Kharif season. So, 1 iscorrect.

Mustard is grown during Rabi season. So, 2 is incorrect.

Learning: The kharif crops include rice, maize, sorghum, pearl millet/bajra, fingermillet/ragi (cereals), arhar (pulses), soyabean, groundnut (oilseeds), cotton etc. Therabi crops include wheat, barley, oats (cereals), chickpea/gram (pulses), linseed,mustard (oilseeds) etc.

Q Source: 9th NCERT: Geography

19 Consider the following statements.Its population is largely found in the North-eastern India.1.It is listed as Endangered in the IUCN Red List.2.It is associated with wet grasslands, swamps and densely vegetated river valleys.3.It feeds on some food crops causing considerable damage to them.4.The above refer to?

White-bellied heronA.Namdapha flying squirrelB.Elvira ratC.Wild Water BuffaloD.

User Answer :Correct Answer : D

INSIG

HTS IAS

Page 13: Insight Test-13 Xaam.in

TEST - 13

User Name : chandan paswan

Total Marks : 200

Mark Scored : 22

(C) Insights Active Learning. | All rights reserved. www.insightsias.com 13

Answer Justification :

Learning: They are native to the Indian Subcontinent and Southeast Asia.

The global population has been estimated at 3,400 individuals, of which about 91%live in India, mostly in Assam. So, 1 is correct.

Wild water buffalos are both diurnal and nocturnal.

Apart from grass, they also eat herbs, fruits, and bark, as well as browsing on treesand shrubs. They also feed on crops, including rice, sugarcane, and jute, sometimescausing considerable damage. So, 4 is correct.

It is included in CITES Appendix III, and is legally protected in Bhutan, India,Nepal, and Thailand.

Q Source: Important Species - Test 13 Syllabus

20 How is the World Bank Group (WBG) President selected?Nominated by the largest shareholder of WBG subject to confirmation by theA.Board of GovernorsElected on a majority basis by the shareholder member states of the WBGB.Elected by the United Nations General Assembly (UNGA) on a two-thirdsC.majority basisAppointed by the UN Secretary General subject to approval by the Board ofD.Governors

User Answer :Correct Answer : AAnswer Justification :

Learning: Traditionally, the Bank President has always been a U.S. citizennominated by the President of the United States, the largest shareholder in the bank.The nominee is subject to confirmation by the Board of Governors, to serve for afive-year, renewable term.

The president is responsible for chairing the meetings of the Boards of Directors andfor overall management of the World Bank Group.

Jim Yong Kim is the President currently.

Q Source: Important International bodies - Test 13 Syllabus

INSIG

HTS IAS

Page 14: Insight Test-13 Xaam.in

TEST - 13

User Name : chandan paswan

Total Marks 200

Mark Scored 22

(C) Insights Active Learning. | All rights reserved. www.insightsias.com 14

21 Consider the following statements about the impeachment of the President of India.Impeachment proceedings can only be started by the Lok Sabha first.1.Nominated members of Lok sabha take part in the impeachment of the President.2.The Supreme Court investigates the charges labeled against the President for3.impeachment.Which of these is /are true?

Only 1A.Only 2B.1 and 3C.Only 3D.

User Answer : DCorrect Answer : BAnswer Justification :

Justification: All the MPs whether elected or nominated take part in the election ofthe President. So, 2 is correct.

Impeachment proceedings can be started by either house of the Parliament. So, 1 iswrong.

The concerned house investigates the charges, not the Supreme Court of India. So, 3is wrong.

Q Source: 11th NCERT: Indian constitution at Work

22 Which of the following was/were the recommendations of the Simon Commission setupin 1927?

India's constitution would be a federal constitution.1.Elections to the legislative assemblies will be based on Universal adult franchise.2.The government of India should have complete control over the high court.3.Separate electorates should be abolished.4.The provincial governments should devolve financial powers to the local bodies.5.Select the correct answer using the codes below.

1, 2 and 4 onlyA.2, 4 and 5 onlyB.1 and 3 onlyC.1, 2, 3, 4 and 5D.

User Answer :Correct Answer : CAnswer Justification :

INSIG

HTS IAS

Page 15: Insight Test-13 Xaam.in

TEST - 13

User Name : chandan paswan

Total Marks : 200

Mark Scored : 22

(C) Insights Active Learning. | All rights reserved. www.insightsias.com 15

Justification: As per the commission, there should be a constitutionalreconstruction in the form of a federal constitution. The provinces should be givenfull autonomy including law.

Other major recommendations include:

The number of members of provincial legislative council should be increased.Governor-general should have complete power to appoint the members of thecabinet.The governor should have discretionary power to relate to internal securityand administrative powers to protect the different communities.The government of India should have complete control over the high court.

Learning: There were no Indian members in the commission. No universalfranchise was proposed and the position of governor-general remained unaffected.

There was no provision to abolish separate electorate but it was rather extended toother communities as well. No financial devolution was proposed.

So, 2, 4 and 5 are incorrect.

Q Source: 10th NCERT: History

23 Availing facilities for instruction in mother-tongue at primary stage is theFundamental Right of a ChildA.Duty of the StateB.Legal right of a ChildC.Constitutional right of a ChildD.

User Answer :Correct Answer : BAnswer Justification :

Justification: Article 350A: It shall be the endeavour of every State and of everylocal authority within the State to provide adequate facilities for instruction in themother-tongue at the primary stage of education to children belonging to linguisticminority groups; and the President may issue such directions to any State as heconsiders necessary or proper for securing the provision of such facilities.

It is a directive principle outside of part IV of the constitution. It is a duty of thestate, and not a right of the citizen.

INSIG

HTS IAS

Page 16: Insight Test-13 Xaam.in

TEST - 13

User Name : chandan paswan

Total Marks 200

Mark Scored 22

(C) Insights Active Learning. | All rights reserved. www.insightsias.com 16

Q Source: 11th NCERT: Indian constitution at Work

24 DRDO has decided to conduct captive flight trials of an advanced Anti-RadiationMissile (ARM). What will ARM be used for?

Targeting enemy's air defence capabilities by attacking radars and communication1.facilitiesAttacking underwater submarines that are clandestinely attached with warships2.Which of the above is/are correct?

1 onlyA.2 onlyB.Both 1 and 2C.NoneD.

User Answer :Correct Answer : AAnswer Justification :

Learning: An anti-radiation missile (ARM) is a missile designed to detect andhome in on an enemy radio emission source.

Typically, these are designed for use against an enemy radar, although jammers andeven radios used for communications can also be targeted in this manner.

The ARM missile will be inducted into Indian Armed Forces till 2018 aftersuccessfully conducting a number of developmental trials. On its induction, Indiawill join other few nations including the US and Germany having ARMs.

Q Source:http://www.thehindu.com/todays-paper/tp-national/captive-flight-trials-of-antiradiation-missile-soon/article8245955.ece

25 Apart from selected MPs, Public Account Committee (PAC) members is/areFinance Secretary, Union Government1.Eminent citizens from Industry and Trade2.Comptroller and Auditor General of India3.Governor, RBI4.Select the correct answer using the codes below.

1 and 3 onlyA.2 and 4 onlyB.1, 2, 3 and 4C.None of the aboveD.

INSIG

HTS IAS

Page 17: Insight Test-13 Xaam.in

TEST - 13

User Name : chandan paswan

Total Marks : 200

Mark Scored : 22

(C) Insights Active Learning. | All rights reserved. www.insightsias.com 17

User Answer :Correct Answer : DAnswer Justification :

Justification:CAG assists the committee; he is not its member. So, 3 is wrong.

RBI Governor may be called upon by the PAC for justification, assistance etc. He isalso not its member. So, 4 is wrong.

The Committee consists of not more than 22 members comprising 15 memberselected by Lok Sabha every year from amongst its members. So, 1 and 2 are wrong.

Learning:

Elections are held according to the principle of proportional representation by meansof single transferable vote and not more than 7 members of Rajya Sabha elected bythat House in like manner are associated with the Committee.

The Chairman is appointed by the Speaker from amongst its members of Lok Sabha.

A Minister is not eligible to be elected as a member of the Committee.

Q Source: 11th NCERT: Indian Constitution at Work

26 What do you understand by the term 'Inflation tax'?Inflation caused by deficit financing which erodes peoples' income thusA.acting as a tax.Low tax rates set by the government leading to high consumption andB.inflationTaxation policy that changes with the rate of inflation in the economyC.None of the aboveD.

User Answer :Correct Answer : AAnswer Justification :

Learning: Inflation tax is not an actual legal tax paid to a government; instead"inflation tax" refers to the penalty for holding cash at a time of high inflation.When the government prints more money or reduces interest rates, it floods themarket with cash, which raises inflation in the long run. If an investor is holdingsecurities, real estate or other assets, the effect of inflation may be negligible. If aperson is holding cash, though, this cash is worth less after inflation has risen.

INSIG

HTS IAS

Page 18: Insight Test-13 Xaam.in

TEST - 13

User Name : chandan paswan

Total Marks 200

Mark Scored 22

(C) Insights Active Learning. | All rights reserved. www.insightsias.com 18

The degree of decrease in the value of cash is termed the inflation tax for the way itpunishes people who hold assets in cash, which tend to be lower- and middle-classwage earners.

It is noteworthy that even deficit financing can act as a tax on people which actuallywas intended to boost demand in the economy.

Q Source: Past year UPSC CSP papers

27 East India Company's monopoly of India trade was abolished by theCharter Act of 1853A.Charter Act of 1813B.Pitt's India ActC.Regulating Act of 1773D.

User Answer : CCorrect Answer : BAnswer Justification :

Learning: It was an Act of the Parliament of the United Kingdom which renewedthe charter issued to the British East India Company, and continued the Company'srule in India.

However, the Company's commercial monopoly was ended, except for the tea tradeand the trade with China. Reflecting the growth of British power in India,

The Act expressly asserted the Crown's sovereignty over British India.It allotted a financial grant to promote education in Indian masses.This act permitted Christian missionaries to propagate English and preachtheir religion.

Q Source: 8th NCERT: History - Part I

28 Consider the following statements about the Chola administration.Assertion (A): Villages were not considered autonomous units during the Chola1.administration.Reason (R): Only those will royal lineage or background were eligible for election2.to village councils which managed the village's affairs.In the context of the above, which of these is correct?

A is correct, and R is an appropriate explanation of A.A.A is correct, but R is not an appropriate explanation of A.B.

INSIG

HTS IAS

Page 19: Insight Test-13 Xaam.in

TEST - 13

User Name : chandan paswan

Total Marks : 200

Mark Scored : 22

(C) Insights Active Learning. | All rights reserved. www.insightsias.com 19

A is incorrect, but R is correct.C.Both A and R are incorrectD.

User Answer : DCorrect Answer : DAnswer Justification :

Justification: The Chola Empire was divided into mandalams and each mandalaminto valanadus and nadus. In each nadu there were a number of autonomous villages.

A village was divided into thirty wards and each was to nominate its members to thevillage council.

The qualifications to become a ward member were:

Ownership of at least one fourth veli of land.Own residence.Above thirty years and below seventy years of age.Knowledge of Vedas.

So, clearly both A and R are wrong.

Q Source: 11th TamilNadu History Textbook

29 Which of the following are the functions of the Reserve Bank of India (RBI)?Stabilizing the rate of inflation1.Performing developmental functions2.Stabilizing excess volatility in the exchange rate of rupee3.Regulating the rate of lending and borrowing via banks in India4.Choose the correct answer using the codes below:

1, 2 and 3A.2, 3 and 4B.1, 3 and 4C.1, 2, 3 and 4D.

User Answer : CCorrect Answer : DAnswer Justification :

Learning: Main functions are:

Monetary Authority: Formulates, implements and monitors the monetarypolicy to maintain price stability and ensuring adequate flow of credit to

INSIG

HTS IAS

Page 20: Insight Test-13 Xaam.in

TEST - 13

User Name : chandan paswan

Total Marks 200

Mark Scored 22

(C) Insights Active Learning. | All rights reserved. www.insightsias.com 20

productive sectors.Regulator and supervisor of the financial system: Prescribes broad parametersof banking operations within which the country's banking and financialsystem functions to maintain public confidence in the system, protectdepositors' interest and provide cost-effective banking services to the public.Manager of Foreign Exchange: Manages the Foreign Exchange ManagementAct, 1999 to facilitate external trade and payment and promote orderlydevelopment and maintenance of foreign exchange market in India.Issuer of currency: Issues and exchanges or destroys currency and coins notfit for circulation.Developmental role: Performs a wide range of promotional functions tosupport national objectives.Related Functions: Banker to the Government: performs merchant bankingfunction for the central and the state governments; also acts as their banker.Banker to banks

Q Source: 12th NCERT: Macroeconomics

30 After consolidating their position in India, the Delhi Sultans introduced certain reformsin the land revenue administration. Concerning it, what were 'Inam Lands'?

Lands assigned to officials as payment for their servicesA.Land from which revenues collected were spent for the maintenance of royalB.court and royal householdLand assigned or granted to religious leaders or religious institutionsC.Land gifted to peasants for increasing production and yield of cropsD.

User Answer : CCorrect Answer : CAnswer Justification :

Justification: Option (a) is similar to Iqta lands.

Option (b) is Khalisa land.

Learning: There were mainly three categories of land - Iqta, Khalisa, Inam and therest given to Zamindars.

An iqta holder was expected to collect the revenue and deduct from in the amountgranted to him, the balance was to be remitted to the central government

The second category of land was the Khalsa, or the royal land. It was under thedirect supervision and control of the government.

INSIG

HTS IAS

Page 21: Insight Test-13 Xaam.in

TEST - 13

User Name : chandan paswan

Total Marks : 200

Mark Scored : 22

(C) Insights Active Learning. | All rights reserved. www.insightsias.com 21

It was probably managed through agents or amils. Another class of land was the onewhich was left with the traditional rajas or Zamindars.

Q Source: 11th TamilNadu History Textbook

31 Which of the following can be the problem(s) associated with the use or consumption ofHard water?

It is a significant health hazard and can worsen the condition of most diseases in1.humans.Dye does not absorb evenly on cloth washed with soap using hard water.2.It can lead to the inefficient operation or failure of water-using appliances.3.Select the correct answer using the codes below.

1 and 2 onlyA.2 and 3 onlyB.1 and 3 onlyC.1, 2 and 3D.

User Answer :Correct Answer : BAnswer Justification :

Justification: Statement 1: Hard water is not a health hazard. In fact, the NationalResearch Council (National Academy of Sciences) states that hard drinking watergenerally contributes a small amount toward total calcium and magnesium humandietary needs. However, it can potentially affect the state of cardiovascular diseases.

Statement 2: A precipitate adheres onto the fibre of the cloth as gummy mass uponwashing with hard water. So, dye is not absorbed easily.

Statement 3: Heated hard water forms a scale of calcium and magnesium minerals(limescale deposits) that can contribute to the inefficient operation or failure ofwater-using appliances. Pipes can become clogged with scale that reduces waterflow

Q Source: 10th NCERT: Science

32 Which of the following statements is INCORRECT with regard to Malwa and Deccanschools of paintings?

Malwa paintings represent a synthesis of the indigenous and the patronizedA.Persian style.Malwa and Deccan were done in miniature styles.B.

INSIG

HTS IAS

Page 22: Insight Test-13 Xaam.in

TEST - 13

User Name : chandan paswan

Total Marks 200

Mark Scored 22

(C) Insights Active Learning. | All rights reserved. www.insightsias.com 22

Both of them were confined to illustrations on books and manuscripts.C.None of the aboveD.

User Answer :Correct Answer : CAnswer Justification :

Learning: The miniature painting style, which flourished initially in the Bahmanicourt and later in the courts of Ahmadnagar, Bijapur and Golkonda is popularlyknown as the Deccan school of Painting.

Malwa is a Central Indian Region where the painting traditions flourished indifferent places right from the pre-Mughal times. Mandu was the main center in theearly era of pre-Mughal times where "Kalpsutra" was painted in 1439 A.D.

Q Source: 11th NCERT: Culture

33 South Asian Free Trade Area (SAFTA) includesBhutan1.Maldives2.Pakistan3.Myamnar4.Afghanistan5.Select the correct answer using the codes below.

All except 4A.All except 5B.All except 1 and 2C.All except 3 and 4D.

User Answer : ACorrect Answer : AAnswer Justification :

Learning: SAFTA in 2004 created a free trade area in Afghanistan, Bangladesh,Bhutan, India, Maldives, Nepal, Pakistan and Sri Lanka.

The foreign ministers of the region signed a framework agreement on SAFTA toreduce customs duties of all traded goods to zero by the year 2016.

The basic principles underlying SAFTA are:

Overall reciprocity and mutuality of advantagesNegotiation of tariff reform

INSIG

HTS IAS

Page 23: Insight Test-13 Xaam.in

TEST - 13

User Name : chandan paswan

Total Marks : 200

Mark Scored : 22

(C) Insights Active Learning. | All rights reserved. www.insightsias.com 23

Recognition of the special needs of the Least Developed Contracting Statesetc.

Q Source: International treaties/agreements ' Test 13 Syllabus

34 Taxation policies affect which of the following components in the Indian economy?Level of domestic investment1.Foreign investment2.Savings3.Inflation4.Select the correct answer using the codes below.

1 and 3 onlyA.2, 3 and 4 onlyB.1and 4 onlyC.1, 2, 3 and 4D.

User Answer : DCorrect Answer : DAnswer Justification :

Justification: Tweaking taxes affect the level of savings in the economy. Savingsaffect investment.

Foreign investments will be affected negatively if corporation and service taxes arevery high in the country. However, investor friendly tax policies such as GST willattract greater domestic as well as foreign investment. So, 1, 2 and 3 are correct.

Increasing taxes in the country will lead to lower disposable income and lesserdemand thus being counter-inflationary. So, 4 is correct.

Q Source: 12th NCERT: Macroeconomics

35 The C. Rajagopalachari's formula was related withPartition of IndiaA.Reform of the Provincial Legislative AssembliesB.Three language formulaC.Reorganization of states on linguistic basisD.

User Answer : ACorrect Answer : AAnswer Justification :

INSIG

HTS IAS

Page 24: Insight Test-13 Xaam.in

TEST - 13

User Name : chandan paswan

Total Marks 200

Mark Scored 22

(C) Insights Active Learning. | All rights reserved. www.insightsias.com 24

Learning: It was a proposal formulated by Chakravarthi Rajagopalachari to solvethe political deadlock between the All India Muslim League and Indian NationalCongress on independence of India from the British.

The League's position was that the Muslims and Hindus of British India wereof two separate nations and hence the Muslims had the right to their ownnation when India obtained independence.He devised a proposal for the Congress to offer the League the MuslimPakistan based on plebiscite of all the people in the regions where Muslimsmade a majority.Although the formula was opposed even within the Congress party, Gandhiused it as his proposal in his talks with Jinnah in 1944. However, Jinnahrejected the proposal and the talks failed.

Q Source: Based on past year UPSC papers ' Qs on C Rajagopalachari

36 Millettia pinnata can be used to produce bio-diesel. Consider the following about it.It is native to India.1.It can be grown in arid regions as well as on high altitudes in Himalayas.2.Wood obtained from the tree is stronger than steel.3.Oil and residue of the plant are very healthy and are generally consumed naturally4.without processing.Select the correct answer using the codes below.

1 and 3 onlyA.2, 3 and 4 onlyB.1 onlyC.1, 2, 3 and 4 onlyD.

User Answer :Correct Answer : CAnswer Justification :

Justification: Statement 1: It is naturally distributed in tropical and temperate Asia,from India to Japan to Thailand to Malesia to north and north-eastern Australia tosome Pacific islands.

Statement 2: It has been propagated and distributed further around the world inhumid and subtropical environments from sea-level to 1200m, although in theHimalayan foothills it is not found above 600m.

Withstanding temperatures slightly below 0 'C and up to about 50 'C and annualrainfall of 50'2,50 cm, the tree grows wild on sandy and rocky soils, including

INSIG

HTS IAS

Page 25: Insight Test-13 Xaam.in

TEST - 13

User Name : chandan paswan

Total Marks : 200

Mark Scored : 22

(C) Insights Active Learning. | All rights reserved. www.insightsias.com 25

oolitic limestone, and will grow in most soil types, even with its roots in salt water.

Statement 3: The wood is said to be beautifully grained but splits easily when sawnthus relegating it to firewood, posts, and tool handles.

Statement 4: While the oil and residue of the plant are toxic and will induce nauseaand vomiting if ingested, the fruits and sprouts, along with the seeds, are used inmany traditional remedies.

Q Source: Based on past year UPSC papers ' Pongamia Pinnata

37 The Brundtland Commission Report 1987 is related toOrganized CrimeA.Nuclear ProliferationB.Sustainable DevelopmentC.International Arm TradeD.

User Answer :Correct Answer : CAnswer Justification :

Learning: Brundtland Report also known as 'Our Common Future' came from theUnited Nations World Commission on Environment and Development (WCED).

The Brundtland Commission Report recognised that human resource development inthe form of poverty reduction, gender equity, and wealth redistribution was crucialto formulating strategies for environmental conservation.

It also recognised that environmental-limits to economic growth in industrialisedand industrialising societies existed.

Q Source: 9th NCERT: Geography

38 A person with zero income may not be able to avail drinking water in a society based onwhich of these economic systems?

Socialistic modelA.Capitalistic modelB.Mixed economic modelC.Communist modelD.

User Answer : BCorrect Answer : B

INSIG

HTS IAS

Page 26: Insight Test-13 Xaam.in

TEST - 13

User Name : chandan paswan

Total Marks 200

Mark Scored 22

(C) Insights Active Learning. | All rights reserved. www.insightsias.com 26

Answer Justification :

Justification: In extreme capitalism, markets are left free without regulation. Onlythose who have the ability to pay can enjoy goods or services.

In such a society, drinking water will also be commodified and sold in the marketbased on the forces of supply and demand. If a person has zero income, he will notbe able to purchase drinking water.

However, this is a hypothetical case, and today many capitalist societies have someor the other form of fundamental social security measures for basic health, water,sanitation and education.

Q Source: 11th NCERT: Indian Economic Development

39 Which of the following practice(s) help stop soil erosion?Mulching1.Mixed farming2.Crop rotation3.Select the correct answer using the codes below.

1 and 2 onlyA.1 onlyB.2 and 3 onlyC.1, 2 and 3D.

User Answer : CCorrect Answer : DAnswer Justification :

Justification: Statement 1: Mulching is when the bare ground between plants iscovered with a layer of organic matter like straw. It helps to retain soil moisture,protects the soil from erosion and reduces compaction from the impact of heavyrains thus stopping soil erosion.

Statement 2: Combining crops and livestock also has the potential to maintainecosystem function and health and help prevent agricultural systems from becomingtoo brittle, or over connected, by promoting greater biodiversity, and thereforeincreased capability to absorb shocks to the natural resource base

Statement 3: Crop rotation allows conservation and rejuvenation of the nutrient baseof soil.

INSIG

HTS IAS

Page 27: Insight Test-13 Xaam.in

TEST - 13

User Name : chandan paswan

Total Marks : 200

Mark Scored : 22

(C) Insights Active Learning. | All rights reserved. www.insightsias.com 27

Q Source: 8th NCERT: Geography

40 Why the Tamil Nadu coast remains dry during the South-west monsoon season?It is located in the Southern most end of India.1.It is situated parallel to the Bay of Bengal branch of southwest monsoon.2.Which of the above is/are correct?

1 onlyA.2 onlyB.Both 1 and 2C.NoneD.

User Answer : BCorrect Answer : BAnswer Justification :

Justification: Since the monsoon winds blow parallel to the coasts, they do notcause precipitation or rain. So, 2 is correct.

The TN coasts also lie in the rainshadow area of the Arabian Sea branch of thesouth-west Monsoon. Thus, they remain largely dry during the summer monsoon.

Even Kerala is located in the southern most end but receives a good amount ofrainfall in the summer monsoon. So, 1 is incorrect.

Q Source: 11th NCERT: India Physical Geography

41 Consider the following statements.Assertion (A): The Himalayan ranges show a succession of vegetation from the1.tropical to the tundra.Reason (R): In mountainous areas, the decrease in temperature with increasing2.altitude leads to a corresponding change in natural vegetation.In the context of the above, which of these is correct?

A is correct, and R may partly explain A.A.A is correct, but R cannot be a reason behind A.B.A is incorrect, but R is correct.C.Both A and R are incorrect.D.

User Answer :Correct Answer : AAnswer Justification :

Learning:Deciduous forests are found in the foothills of the Himalayas. It is

INSIG

HTS IAS

Page 28: Insight Test-13 Xaam.in

TEST - 13

User Name : chandan paswan

Total Marks 200

Mark Scored 22

(C) Insights Active Learning. | All rights reserved. www.insightsias.com 28

succeeded by the wet temperate type of forests between an altitude of 1,000-2,000m.

In the higher hill ranges of northeastern India, hilly areas of West Bengal andUttaranchal, evergreen broad leaf trees such as oak and chestnut are predominant.

Between 1,500-1,750 m, pine forests are also well-developed in this zone, with ChirPine as a very useful commercial tree

Q Source:11th NCERT: India Physical Geography

42 The single largest component of the annual budget under revenue expenditure generallyis

Defence ServicesA.Interest Payments and Prepayment PremiumB.Grants to State governments and Union Territories (UTs)C.PensionsD.

User Answer : CCorrect Answer : BAnswer Justification :

Learning:Please refer to http://indiabudget.nic.in/ub2015-16/bag/bag3.pdfto see allmajor heads under Revenue Expenditure.

Receipts are herehttp://indiabudget.nic.in/ub2015-16/bag/bag2.pdf

Focus on other components too, for e.g. taxes. Many budget based questions areasked in UPSC.

Q Source:12th NCERT: Macroeconomics and annual budget

43 Consider the following about the National Legal Services Authority (NALSA).It is a statutory body headed by the Union Minister for Law and Justice.1.It organizes Lok Adalats for speedy resolution of cases.2.It provides free legal services only to disabled citizens or those Below Poverty Line3.(BPL).A District Legal Services Authority is constituted in every District.4.Select the correct answer using the codes below.

1 and 3 onlyA.2 and 4 onlyB.

INSIG

HTS IAS

Page 29: Insight Test-13 Xaam.in

TEST - 13

User Name : chandan paswan

Total Marks : 200

Mark Scored : 22

(C) Insights Active Learning. | All rights reserved. www.insightsias.com 29

2, 3 and 4 onlyC.1, 2, 3 and 4D.

User Answer :Correct Answer : BAnswer Justification :

Justification:Statement 1 and 4: It is headed by the Chief Justice of India.

There is a provision for similar mechanism at state and district level also headed byChief Justice of High Courts and Chief Judges of District courts respectively.

Statement 3: Eligible persons for getting free legal services

Women and children;Members of SC/STIndustrial workmenVictims of mass disaster, violence, flood, drought, earthquake, industrialdisaster.Disabled persons.Person in custodyPersons whose annual income does not exceed Rs. 1,00,000/-

Q Source:Past year UPSC papers

44 Consider the following about the Karbi tribe.The Karbis are known for their Matrilineal society.1.These tribes largely follow Animism.2.Some members of the tribe live in areas administered under the Sixth Schedule of3.the constitution.Select the correct answer using the codes below.

1 and 2 onlyA.2 and 3 onlyB.1 and 3 onlyC.1, 2 and 3D.

User Answer :Correct Answer : BAnswer Justification :

Justification:Statement 1: The Karbis are a Patrilineal society and is composed offive major clans which are further sub-divided into sub-clans.

INSIG

HTS IAS

Page 30: Insight Test-13 Xaam.in

TEST - 13

User Name : chandan paswan

Total Marks 200

Mark Scored 22

(C) Insights Active Learning. | All rights reserved. www.insightsias.com 30

Statement 2: Animism is the worldview that non-human entities'such as animals,plants, and inanimate objects'possess a spiritual essence

Statement 3: The Karbis are the principal tribal community in the Karbi Anglongdistrict of Assam, a district administered as per the provisions of the Sixth Scheduleof the Constitution of India, having an autonomous district of their own since 1951.

Q Source: Past year UPSC papers

45 Which among the following industries is the largest consumer of water in India?FertilizerA.SugarB.Thermal power plantsC.Pulp and PaperD.

User Answer : CCorrect Answer : CAnswer Justification :

Learning:Most TPPs in India are owned by the government. Indian TPPs are one ofthe highest consumers of water as compared to their global counterparts. On anaverage, for every 1000 Kwh power, Indian TPPs consume as much as 80 cubicmeters of water. The water consumption in the modern TPPs in developed countriesis less than 10 cubic meters for every 1000 Kwh.

Read this to know about water usage industry wise and nation wise.

http://www.cseindia.org/dte-supplement/industry20040215/misuse.htm

Q Source:UPSC past year papers

46 Ganges shark is critically endangered becauseThey need to migrate to salt water to reproduce offsprings.1.They are misinterpreted as large catfish and killed as they damage local ecosystem.2.Which of the above is/are correct?

1 onlyA.2 onlyB.Both 1 and 2C.NoneD.

User Answer : BCorrect Answer : D

INSIG

HTS IAS

Page 31: Insight Test-13 Xaam.in

TEST - 13

User Name : chandan paswan

Total Marks : 200

Mark Scored : 22

(C) Insights Active Learning. | All rights reserved. www.insightsias.com 31

Answer Justification :

Justification:Statement 1: It is bull sharks that migrate to salt water to reproduce,not Ganges shark. Ganges shark is found in a very narrow range of habitat. So, 1 iswrong.

However, construction of dams and barrages have affected their migratory patternsand threatened their life cycles.

Statement 2: It is wrong. They are misterpreted for bull sharks. Catfishes look verydifferent from this shark. Overfishing, habitat degradation from pollution, increasingriver use and management, local killing for meat, gillnet and oil etc are reasons fortheir population decline.

Q Source: Species mentioned in previous year UPSC CSP questions

47 Consider the following statements.Assertion (A): The storage life of a fresh fruit is influenced by its respiratory1.activity.Reason(R): Metabolic activity in fresh fruits continues for a short period even after2.harvest.In the context of the above, which of these is correct?

A is correct, and R is an appropriate explanation of A.A.A is correct, but R cannot be an appropriate explanation of A.B.A is incorrect, but R is correct.C.Both A and R are incorrect.D.

User Answer :Correct Answer : AAnswer Justification :

Justification & Learning:Respiration is the chemical process by which fruits andvegetables convert sugars and oxygen into carbon dioxide, water, and heat.Metabolic activity in fresh fruits and vegetables continues for a short period afterharvest.

The energy required to sustain this activity comes from the respiration process.

Respiration involves the oxidation of sugars to produce carbon dioxide, water andheat. The storage life of a commodity is influenced by its respiratory activity.

By storing a commodity at low temperature, respiration is reduced and senescence is

INSIG

HTS IAS

Page 32: Insight Test-13 Xaam.in

TEST - 13

User Name : chandan paswan

Total Marks 200

Mark Scored 22

(C) Insights Active Learning. | All rights reserved. www.insightsias.com 32

delayed, thus extending storage life.

Q Source:Improvisation: UPSC CSP papers

48 Consider the following statements.When the moon's orbit is closest to the earth, tidal range is lower than normal.1.When the earth is closest to the Sun tidal range is greater than normal.2.Which of the above is/are correct?

1 onlyA.2 onlyB.Both 1 and 2C.NoneD.

User Answer : DCorrect Answer : BAnswer Justification :

Justification:Tidal range is affected by the gravitational forces of the Sun andMoon. The closer they are to earth, higher is the tidal range.

When earth is closest to both Sun and Moon, unusually high tides occur. It alsooccurs when earth is closest to any one of these.

Q Source:11th NCERT: Fundamentals of Physical Geography

49 If RBI wishes to increase money supply in Indian economy, it wouldDecrease Reverse repo rate1.Increase Cash Reserve Ratio (CRR)2.Issue orders to public to keep all private financial savings in bank accounts3.Which of the above is/are correct?

1 and 2 onlyA.2 and 3 onlyB.1 and 3 onlyC.1 onlyD.

User Answer : DCorrect Answer : DAnswer Justification :

Justification:Statement 1: Reverse repo rate is basically the rate that banks get forparking their funds with the RBI. Reducing this rate makes the bank park theirmoney somewhere else and not with RBI. For it is less attractive to them. Money

INSIG

HTS IAS

Page 33: Insight Test-13 Xaam.in

TEST - 13

User Name : chandan paswan

Total Marks : 200

Mark Scored : 22

(C) Insights Active Learning. | All rights reserved. www.insightsias.com 33

parked outside RBI circulated by the banks would become a part of the moneysupply.

Statement 2: Increasing CRR sucks liquidity from the banking system and increasesthe idle deposits of banks with RBI.

Statement 3: Banking deposits and the money held by the public form a part of themoney supply. Even if a customer deposits money in bank, it will not change thetotal money supply, but only its liquidity.

Q Source:12th NCERT: Macroeconomics

50 Consider the following statements about rock formation.Magnetically susceptible minerals get aligned to the earth's magnetic field during1.the period of rock formation.Foliated metamorphic rocks are formed within the Earth's interior under extremely2.high pressures that are unequal in different directions.Which of the above is/are correct?

1 onlyA.2 onlyB.Both 1 and 2C.NoneD.

User Answer :Correct Answer : CAnswer Justification :

Justification: Statement 1: This comes due to geomagnetism. Magnetic response ofrocks is determined by amounts and susceptibilities of constituent minerals.Sedimentary rocks have very low susceptibility to magnetic field of earth. So, thereis little alignment in their constituents in response to earth's magnetic field. Forultramafic rocks it is the highest.

Statement 2: Foliation refers to repetitive layering in metamorphic rocks.

When the pressure is greater in one direction than in the others (directed pressure), itcauses the minerals in the original rock to reorient themselves with the long and flatminerals aligning perpendicular to the greatest pressure direction. This reduces theoverall pressure on the rock and gives it a stripped look.

Q Source: Geomagnetism and rock formation topics in 11th NCERT: Fundamentalsof Physical Geography

INSIG

HTS IAS

Page 34: Insight Test-13 Xaam.in

TEST - 13

User Name : chandan paswan

Total Marks 200

Mark Scored 22

(C) Insights Active Learning. | All rights reserved. www.insightsias.com 34

51 Katabatic winds occur due toDensity differences in wind at different altitudesA.Differential warming of the ocean surface near coastsB.Local thunderstorms guided by high convection ratesC.Seasonal land breeze in the interior of continentsD.

User Answer :Correct Answer : AAnswer Justification :

Learning: These are downslope winds flowing from high elevations of mountains,plateaus, and hills down their slopes to the valleys or planes below.

A katabatic wind originates from radiational cooling of air atop a plateau, amountain, glacier, or even a hill.

Since the density of air is inversely proportional to temperature, the air will flowdownwards, warming approximately adiabatically as it descends.

The temperature of the air depends on the temperature in the source region and theamount of descent.

Q Source: 11th NCERT: Fundamentals of Physical Geography ' Glossary

52 High sulphur dioxide in air has which of the following harmful effects?It can cause irritation to the eyes resulting in tears and redness.1.Its high concentration leads to stiffness of flower buds and their falling off from2.plants.Which of the above is/are correct?

1 onlyA.2 onlyB.Both 1 and 2C.NoneD.

User Answer :Correct Answer : CAnswer Justification :

Justification & Learning: Sulphur dioxide irritates the skin and mucousmembranes of the eyes, nose, throat, and lungs. So, 1 is correct.

High concentrations of SO2 can cause inflammation and irritation of the respiratorysystem, particularly during heavy physical activity. The resulting symptoms may

INSIG

HTS IAS

Page 35: Insight Test-13 Xaam.in

TEST - 13

User Name : chandan paswan

Total Marks : 200

Mark Scored : 22

(C) Insights Active Learning. | All rights reserved. www.insightsias.com 35

include pain when taking a deep breath, coughing, throat irritation, and breathingdifficulties.

Q Source: 11th Science NCERT: Chapter 14: Environmental Chemistry

53 Lead can be found inPaints1.Batteries2.Some colours of Ink3.Exhaust emissions4.Select the correct answer using the codes below.

1 and 2 onlyA.1, 3 and 4 onlyB.2 and 4 onlyC.1, 2, 3 and 4D.

User Answer :Correct Answer : DAnswer Justification :

Learning: Lead was used in paint to add colour, improve the ability of the paint tohide the surface it covers, and to make it last longer.

Lead seldom occurs naturally in water supplies like rivers and lakes. Lead entersdrinking water primarily as a result of the corrosion, or wearing away, of materialscontaining lead in the water distribution system and household or building plumbing

Q Source: Past year UPSC CSP papers

54 Rainfall is more over the oceans than on the landmasses of the world becauseOceans receive larger solar insolation per unit area than landA.Ocean water is salty.B.Oceans are huge sources of water as compared to land.C.Frequent cyclones occur over the oceans as compared to land.D.

User Answer : DCorrect Answer : CAnswer Justification :

Justification: Option (a): Oceans receive lesser solar insolation per unit area due tothe cloudiness over them. Clouds reflect back solar insolation.

INSIG

HTS IAS

Page 36: Insight Test-13 Xaam.in

TEST - 13

User Name : chandan paswan

Total Marks 200

Mark Scored 22

(C) Insights Active Learning. | All rights reserved. www.insightsias.com 36

Option (d): Cyclones occur due to intense evaporation and existence of low pressurezone. They are one of the phenomena associated with high rainfall on oceans, not itscause.

Since there is more water in oceans and wind movement is unable to carry allcloudiness to land, rainfall is greater in oceans.

Q Source: 11th NCERT: Fundamentals of Physical Geography

55 Consider the following statements about the legislative process in the states.The status of legislative councils in passing ordinary bills is weaker than Rajya1.Sabha.There is no provision of joint sitting in case of a disagreement between the two2.houses in the states.If the Governor accepts the changes made by the legislative council which were not3.accepted by the assembly, then the bill is deemed to be passed.Select the correct answer using the codes below.

1 and 2 onlyA.2 and 3 onlyB.1 and 3 onlyC.All of the aboveD.

User Answer : ACorrect Answer : AAnswer Justification :

Justification: The Governor cannot accept a bill which has not been passed by theassembly. Doing so would be unconstitutional. So, 3 is incorrect.

Joint sitting provision is not there, because the assembly has overriding powers withrespect to passing any bill in the state. So, the Council is much weaker than theassembly. Thus, 1 and 2 are correct.

Q Source: 11th NCERT: Indian constitution at Work

56 The early atmosphere on earth in the course of its evolution largely containedWater vapour, nitrogen, carbon dioxide, methane, ammonia and very little ofA.free oxygenAbundant oxygen, water vapour, carbon dioxide, methane and ammoniaB.Carbon dioxide, methane and very little of free ammonia and nitrogenC.Sulphur dioxide, carbon dioxide, methane, water vapour and very little of freeD.

INSIG

HTS IAS

Page 37: Insight Test-13 Xaam.in

TEST - 13

User Name : chandan paswan

Total Marks : 200

Mark Scored : 22

(C) Insights Active Learning. | All rights reserved. www.insightsias.com 37

ammoniaUser Answer :Correct Answer : AAnswer Justification :

Learning: During the cooling of the earth, gases and water vapour were releasedfrom the interior solid earth.

This started the evolution of the present atmosphere. The early atmosphere largelycontained water vapour, nitrogen carbon dioxide, methane, ammonia and very littleof free oxygen.

The process through which the gases were outpoured from the interior is calleddegassing. Continuous volcanic eruptions contributed water vapour and gases.

Q Source: 11th NCERT: Fundamentals of Physical Geography

57 Consider the following statements.Its topography supports scrub-thorn arid forests, rocky landscapes, dry deciduous1.forests, rocks, grasses and hilly cliffs.Some of the rarest feathered species like grey partridge, white-throated kingfisher2.and Indian peafowl are found here.It is the location of several sites of historical importance like the 16th-century3.Kankwadi fort.The above refer to?

Sariska Tiger ReserveA.Simlipal National ParkB.Jim Corbett National ParkC.Ranthambore National ParkD.

User Answer :Correct Answer : AAnswer Justification :

Learning: It is located in the Alwar district of the state of Rajasthan.

The area of Sariska, being a part of the Aravalli Range, is rich in mineral resources,such as copper. In spite of the Supreme Court's 1991 ban on mining in the area,marble mining continues to threaten the environment.

The dominant tree in the forests is dhok.

INSIG

HTS IAS

Page 38: Insight Test-13 Xaam.in

TEST - 13

User Name : chandan paswan

Total Marks 200

Mark Scored 22

(C) Insights Active Learning. | All rights reserved. www.insightsias.com 38

Pandupol in the hills in the centre of the reserve is believed to be one of the retreatsof Pandavas.

Q Source: Test 13 Syllabus ' National Parks

58 Which of these organism(s) migrate in search of more hospitable climates?Insects1.Mammals2.Fishes3.Select the correct answer using the codes below.

1 and 2 onlyA.2 onlyB.2 and 3 onlyC.1, 2 and 3D.

User Answer : CCorrect Answer : DAnswer Justification :

Justification & Learning: Statement 1: Dragonflies, beetles, butterflies and mothsare common insect migrants. In some cases the individuals that migrate in onedirection may not return and the next generation may instead migrate in the oppositedirection. This is a significant difference from bird migration.

Statement 2: Gazelles, Zebra, Whales, Dolphin are some common mammalmigrants. Mass migration occurs in mammals with a kind of annual circular patternof movement in many cases.

Statement 3: Most fish species are relatively limited in their movements, remainingin a single geographical area and making short migrations for wintering, to spawn,or to feed. A few hundred species migrate long distances, in some cases ofthousands of kilometres. About 120 species of fish, including several species ofsalmon, migrate between saltwater and freshwater.

Q Source: 6th Science NCERT

59 At the international level, Geographical Indication (GI) tag is governed by theTrade-Related Aspects of Intellectual Property Rights (TRIPS)A.Agreement on Trans-national Resources (AOTNR)B.International Treaty on Plant Genetic Resources for Food and AgricultureC.World Heritage List, UNESCOD.

INSIG

HTS IAS

Page 39: Insight Test-13 Xaam.in

TEST - 13

User Name : chandan paswan

Total Marks : 200

Mark Scored : 22

(C) Insights Active Learning. | All rights reserved. www.insightsias.com 39

User Answer :Correct Answer : AAnswer Justification :

Learning: Geographical Indication (GI) Geographical Indication is an insignia onproducts having a unique geographical origin and evolution over centuries withregards to its special quality or reputation attributes.

It is a mark of authenticity and ensures that registered authorised users (or at leastthose residing inside the geographic territory) are allowed to use the popular productname, e.g. Hyderabadi Biryani.

At international level, GI is governed by World Trade Organisation's(WTO's) Agreement on Trade-Related Aspects of Intellectual Property Rights(TRIPS).TRIPS requires WTO members to provide copyright rights, covering contentproducers including performers, producers of sound recordings andbroadcasting organizations; geographical indications, including appellationsof origin; industrial designs; integrated circuit layout-designs; patents; newplant varieties; trademarks etc.In India, GI registration is governed by the Geographical Indications of goods(Registration and Protection) Act, 1999. Darjeeling tea was the first productin India accorded with GI tag.

Q Source: Frequently in News and CSP 2015

60 Weak muscles and feeling very little energy to work is a symptom of the deficiency ofVitamin AA.IodineB.Vitamin B1C.Vitamin KD.

User Answer : BCorrect Answer : CAnswer Justification :

Justification: Vitamin A deficiency can cause vision related problems, even loss ofvision sometimes. So, (a) is incorrect.

Iodine deficieny causes goitre and mental disability in children. So, (b) is incorrect.

Vitamin B1, also called as Thiamine, deficiency may lead to chronic diseases such

INSIG

HTS IAS

Page 40: Insight Test-13 Xaam.in

TEST - 13

User Name : chandan paswan

Total Marks 200

Mark Scored 22

(C) Insights Active Learning. | All rights reserved. www.insightsias.com 40

as Beriberi which is a neurological and cardiovascular disease. Muscle weaknessand energy deficit in body etc are other symptoms associated with its deficiency.

Very small amounts of vitamin B1 are found in virtually all foods.

Q Source: 6th Science NCERT

61 Consider the following about the powers of the Speaker, Lok Sabha.She prorogues an ongoing session of the Lok Sabha.1.She approves all the bills before it is sent for Presidential assent.2.She heads the appointments committee of the Cabinet and in Lok Sabha.3.Select the correct answer using the codes below.

2 onlyA.1 and 3 onlyB.3 onlyC.None of the aboveD.

User Answer :Correct Answer : DAnswer Justification :

Justification: President prorogues the session, i.e. ends it. Speaker can only adjournit for some time. So, 1 is wrong.

She only notifies whether a bill is money bill or not.2 is wrong.

The PM heads the appointments committee of the Cabinet. There is no appointmentscommittee in the Lok Sabha. So, 3 is wrong.

Q Source: 11th NCERT: Indian Constitution at work

62 What do you understand by the term 'Hidden Hunger'?Deficiency of micronutrients, proteins and vitamins despite getting a calorieA.rich dietPercentage of hungry people that go unaccounted in official surveysB.A phenomenon that exists in social classes that are economically well off butC.lack in nutrition and health related aspectsMalnourishment of foetus during pregnancy of weak and malnourishedD.mothers

User Answer :Correct Answer : A

INSIG

HTS IAS

Page 41: Insight Test-13 Xaam.in

TEST - 13

User Name : chandan paswan

Total Marks : 200

Mark Scored : 22

(C) Insights Active Learning. | All rights reserved. www.insightsias.com 41

Answer Justification :

Learning: More than 840 million people in the world do not have adequate food tomeet their daily food and nutritional requirements.

A far greater number' three billion people ' suffer from micronutrient, protein andvitamin deficiencies or 'hidden hunger' because they cannot afford to buy enoughfruits, vegetables, legumes, fish and meat.

Diets lacking essential micronutrients ' particularly iron, vitamin A, iodine and zinc 'increase the risk for disease, reduce lifespan and reduce mental abilities.

Q Source: Frequently in news and asked in past year UPSC papers

63 Which of the following can be the reason(s) for greater biological productivity at thetropics as compared to the higher latitudes?

Higher solar energy is available at tropicsA.Less extreme seasonal variation as compared to the higher latitudesB.Remained undisturbed for longer durations than higher latitudesC.All of the aboveD.

User Answer :Correct Answer : DAnswer Justification :

Justification: Option (a): There is more solar energy available in the tropics, whichcontributes to higher productivity; this in turn might contribute indirectly to greaterdiversity.

Option (b): Tropical environments, unlike temperate ones, are less seasonal,relatively more constant and predictable. Such constant environments promote nichespecialisation and lead to a greater species diversity and

Option (c): Speciation is generally a function of time, unlike temperate regionssubjected to frequent glaciations in the past, tropical latitudes have remainedrelatively undisturbed for millions of years and thus, had a long evolutionary timefor species diversification.

Q Source: 12th Biology NCERT

64 A constitutional amendment bill

INSIG

HTS IAS

Page 42: Insight Test-13 Xaam.in

TEST - 13

User Name : chandan paswan

Total Marks 200

Mark Scored 22

(C) Insights Active Learning. | All rights reserved. www.insightsias.com 42

Can only be initiated by the President of India1.Must be introduced in the Lok Sabha first2.Is subject to the casting vote of the Chairman of the house3.Cannot be introduced by private MPs4.Select the correct answer using the codes below.

1, 2 and 4 onlyA.1, 2 and 3 onlyB.4 onlyC.None of the aboveD.

User Answer : DCorrect Answer : CAnswer Justification :

Justification: Statement 1: Only certain category of CA bills need the priorpermission of the President, for e.g. major tax reforms.

Statement 2: It can be introduced in either house.

Statement 3: Speaker has the casting vote only in case of a tie or undecisivesituations. Otherwise, if the MPs clear the bill, it is sent to the President.

Statement 4: Any MP can introduce a CA bill.

Q Source: 11th NCERT: Indian constitution at Work

65 What is/are the remedies available to a common citizen against the excesses of theexecutive branch of Government?

Get the executive order declared void after a Judicial Review1.Appeal to the High Court under Article 226 of the Constitution2.Approach tribunals made for the specific purpose3.Select the correct answer using the codes below.

1and 2 onlyA.1 and 3 onlyB.2 onlyC.1, 2 and 3D.

User Answer :Correct Answer : DAnswer Justification :

Justification: Article 13 gives Judiciary the power to review legislative acts andadministrative rules. So, 1 is correct.

INSIG

HTS IAS

Page 43: Insight Test-13 Xaam.in

TEST - 13

User Name : chandan paswan

Total Marks : 200

Mark Scored : 22

(C) Insights Active Learning. | All rights reserved. www.insightsias.com 43

Article 32 and 226 give judiciary the power to correct excesses that have led to theviolation of the fundamental rights of a citizen by the state. So, 2 is correct.

Tribunals are established under Article 323 of the Constitution for granting reliefand settling disputes regarding taxation, competitive practices, service matters etc.So, 3 is correct.

Q Source: 11th NCERT: Indian constitution at Work

66 The Union Cabinet has approved a proposal to notify the Trade Facilitation Agreement(TFA) of the World Trade Organization (WTO). Consider the following about the TFA.

It is the WTO's first-ever multilateral accord that aims to simplify customs1.regulations for the cross-border movement of goods.It will make it easier for developing countries to trade with the developed world in2.global markets.It was an outcome of the Bali package.3.Select the correct answer using the codes below.

1 and 2 onlyA.2 and 3 onlyB.3 onlyC.1, 2 and 3D.

User Answer :Correct Answer : DAnswer Justification :

Learning: Developed countries would abolish hard import quotas on agriculturalproducts from the developing world and instead would only be allowed to chargetariffs on amount of agricultural imports exceeding specific limits.

It also aims to reduce red-tapism to facilitate trade by reforming customsbureaucracies and formalities.

Q Source: http://pib.nic.in/newsite/PrintRelease.aspx?relid=136486

67 Consider the following statements.Assertion (A): The Governor exercises more discretion that the President does in1.their respective spheres of influences.Reason (R): The constitutional provisions explicitly limit the overall discretion of2.the President, which is not so clearly defined for the Governor.In the context of the above, which of these is correct?

INSIG

HTS IAS

Page 44: Insight Test-13 Xaam.in

TEST - 13

User Name : chandan paswan

Total Marks 200

Mark Scored 22

(C) Insights Active Learning. | All rights reserved. www.insightsias.com 44

A is correct, and R is an appropriate explanation of A.A.A is correct, but R is not an appropriate explanation of A.B.A is incorrect, but R is correct.C.Both A and R are incorrect.D.

User Answer :Correct Answer : AAnswer Justification :

Justification: Article 74(1) clearly prescribes that the President is bound by theadvice of the council of Ministers. However, in the case of Governor, the respectiveprovision does not make the advice of the council of ministers in state binding onhim.

Article 163 is clear in this regard:

There shall be a Council of Ministers with the Chief Minister at the head toaid and advise the Governor in the exercise of his function, except in so far ashe is by or under this Constitution required to exercise his functions or any ofthem in his discretion.If any question arises whether any matter is or is not a matter as respectswhich the Governor is by or under this Constitution required to act in hisdiscretion, the decision of the Governor in his discretion shall be final, andthe validity of anything done by the Governor shall not be called in questionon the ground that he ought or ought not to have acted in his discretion.

Q Source: 11th NCERT: Indian constitution at Work

68 Consider the following about the administration of Jammu and Kashmir (J&K).No emergency due to internal disturbances can be declared in J&K without the1.concurrence of the State.The union government cannot impose a financial emergency in the State.2.Amendments to the Indian Constitution can only apply in concurrence with the3.government of J&K.The Government of India is responsible for the external security of J&K.4.Select the correct answer using the codes below.

1, 2 and 3 onlyA.2, 3 and 4 onlyB.1 and 4 onlyC.1, 2, 3 and 4D.

User Answer :

INSIG

HTS IAS

Page 45: Insight Test-13 Xaam.in

TEST - 13

User Name : chandan paswan

Total Marks : 200

Mark Scored : 22

(C) Insights Active Learning. | All rights reserved. www.insightsias.com 45

Correct Answer : DAnswer Justification :

Justification: According to Article 370, the concurrence of the State is required formaking any laws in matters mentioned in the Union and Concurrent lists.

The remaining differences between the other States and the State of J&K are that noemergency due to internal disturbances can be declared in J&K without theconcurrence of the State.

The union government cannot impose a financial emergency in the State and theDirective Principles do not apply in J&K.

Finally, amendments to the Indian Constitution (under Art. 368) can only apply inconcurrence with the government of J&K.

Q Source: 11th NCERT: Indian constitution at Work

69 Consider the following statements about the Directive Principles of State Policy(DPSP).

The inheritance of DPSP in the Indian constitution is British colonial legacy.1.Legislations or rules are always required for implementing DPSP.2.Which of the above is/are correct?

1 onlyA.2 onlyB.Both 1 and 2C.NoneD.

User Answer : DCorrect Answer : CAnswer Justification :

Justification: DPSP is not self-enforceable. It requires the parliament to make lawsto implement them.

They were inherited from the British Instrument of instructions given to thegovernors and governor-generals under the Government of India Act 1935.

Q Source: 11th NCERT: Indian constitution at Work

70 Under WTO's provision of 'aggregate measure of support', which of the following

INSIG

HTS IAS

Page 46: Insight Test-13 Xaam.in

TEST - 13

User Name : chandan paswan

Total Marks 200

Mark Scored 22

(C) Insights Active Learning. | All rights reserved. www.insightsias.com 46

would qualify?Subsidies on Irrigation1.Subsidies on Power2.Credit subsidies3.Minimum Support Price (MSP) regime4.Select the correct answer using the codes below:

3 and 4 onlyA.1, 2 and 3 onlyB.4 onlyC.1, 2, 3 and 4 onlyD.

User Answer :Correct Answer : DAnswer Justification :

Learning: WTO agreement envisages two kinds of support to agriculture, viz.domestic support and export subsidies.

The domestic support is further classified into five categories: (a) aggregate measureof support (AMS) which includes product specific and non-product specific support(b) green box support (c) blue box support (d) de minimus support and (e) specialand differential (S&D) treatment box.

Out of these, WTO agreement requires reduction only in AMS and export subsidies,whereas, support under all other heads is exempted.

AMS includes (a) sum total of subsidies on inputs like fertiliser, water, credit, poweretc and (b) market price support measured by calculating the difference betweendomestic administered market price and external reference price (world price)multiplied by quantity of production eligible to get applied administered price.

Q Source: Past year UPSC CSP papers

71 Cripps mission was sent to India toDiscuss the modalities of partition of IndiaA.Negotiate Indian National Congress' (INC) stand in the Round TableB.ConferencesAddress the issue of war crimes and extra-judicial killings by the BritishC.Secure full Indian cooperation and support for British efforts in World WarD.II.

User Answer :Correct Answer : D

INSIG

HTS IAS

Page 47: Insight Test-13 Xaam.in

TEST - 13

User Name : chandan paswan

Total Marks : 200

Mark Scored : 22

(C) Insights Active Learning. | All rights reserved. www.insightsias.com 47

Answer Justification :

Background:The British feared that the destabilizing of India might encourage aJapanese invasion, and would reduce the number of men who volunteered to fightthe war.

Japan in 1942 had overrun Malaya and was into Burma; the threat of an invasion ofIndia was real.

British wanted the cooperation and support of Indian political leaders in order torecruit more Indians into the British Indian Army, which was fighting in the MiddleEast theatre.

Mission:

The war cabinet of Britain sent Sir Stafford Cripps to India in 1942 to elicitcooperation from the Indians.It promised for the fulfillment of past promises to self government to Indianpeople.The proposal of the Cripps mission was that: "India would be a dominionassociated with the United kingdom".

It promised that immediately after the war is stopped, steps would be taken up to setup an elected body charged with the task of making the constitution for India andprovisions would be made so that the Indian states could participate in the framingof the constitution.

Q Source: 10th NCERT: History

72 In the International Monetary Fund (IMF), the vote of each country is weighed by itsMoney contribution to the IMFA.Size of GDPB.Diplomatic and military mightC.An index weighting all of (a), (b) and (c)D.

User Answer : BCorrect Answer : AAnswer Justification :

Learning: International Monetary Fund (IMF) is one of the biggest moneylendersfor any country in the world.

INSIG

HTS IAS

Page 48: Insight Test-13 Xaam.in

TEST - 13

User Name : chandan paswan

Total Marks 200

Mark Scored 22

(C) Insights Active Learning. | All rights reserved. www.insightsias.com 48

Its 188 member states (as in 2012) do not have equal voting rights. The vote of eachcountry is weighed by how much money it has contributed to the IMF.

Majority of the voting power in the IMF is in the hands of only ten countries (US,Japan, Germany, France, UK, China, Italy, Saudi Arabia, Canada and Russia). Theremaining 178 countries have very little say in how these international organisationstake decisions. The World Bank has a similar system of voting.

Q Source: Important International bodies - Test 13 Syllabus

73 Consider the following about Regional Rural Banks (RRBs).They have been established by an Act of Parliament.1.It does not receive any grant or support from the government and is sponsored by a2.parent commercial bank.RRBs operate in all districts in India.3.RRBs are allowed to mobilize deposits only from rural areas.4.Select the correct answer using the codes below.

3 and 4 onlyA.2, 3 and 4 onlyB.1 onlyC.1, 2 and 3 onlyD.

User Answer :Correct Answer : CAnswer Justification :

Justification:Regional Rural Banks (RRBs) were established in 1975 under theprovisions of an Ordinance and followed by Regional Rural Banks Act, 1976. So, 1is correct.

It was done with a view to develop the rural economy and to create a supplementarychannel to the 'Cooperative Credit Structure' with a view to enlarge institutionalcredit for the rural and agriculture sector.

The Government of India, the concerned State Government and the bank, which hadsponsored the RRB contributed to the share capital of RRBs in the proportion of50%, 15% and 35%, respectively. So, 2 is incorrect.

The area of operation of the RRBs is limited to notified few districts in a State. So, 3is also incorrect.

The RRBs mobilise deposits primarily from rural/semi-urban areas and provide

INSIG

HTS IAS

Page 49: Insight Test-13 Xaam.in

TEST - 13

User Name : chandan paswan

Total Marks : 200

Mark Scored : 22

(C) Insights Active Learning. | All rights reserved. www.insightsias.com 49

loans and advances mostly to small and marginal farmers, agricultural labourers,rural artisans and other segments of priority sector. So, 4 is incorrect too.

Q Source: RRBs are asked frequently in UPSC questions

74 Consider the following statements in relation to the Article 371-A of the constitutionmaking special provisions for Nagaland.

The Nagaland state assembly can declare that a law made by the Parliament is not1.applicable to it on religious and social grounds.The Nagaland state assembly can override the laws of the Parliament concerning2.transfer of land and its resources.Which of the above is/are correct?

1 onlyA.2 onlyB.Both 1 and 2C.NoneD.

User Answer :Correct Answer : CAnswer Justification :

Justification:As per Article 371 (1), notwithstanding anything in this Constitution,no Act of Parliament in respect of

religious or social practices of the Nagas,Naga customary law and procedure,administration of civil and criminal justice involving decisions according toNaga customary law,ownership and transfer of land and its resources,

shall apply to the State of Nagaland unless the Legislative Assembly of Nagaland bya resolution so decides.

Q Source: Naga affairs are frequently in news and 11th NCERT: Indian constitutionat Work

75 Consider the following statements.Tropical cyclones originate only over the seas.1.In the tropical latitudes, tropical cyclones generally move from east to west.2.Which of the above is/are correct?

1 onlyA.

INSIG

HTS IAS

Page 50: Insight Test-13 Xaam.in

TEST - 13

User Name : chandan paswan

Total Marks 200

Mark Scored 22

(C) Insights Active Learning. | All rights reserved. www.insightsias.com 50

2 onlyB.Both 1 and 2C.NoneD.

User Answer :Correct Answer : CAnswer Justification :

Justification:The tropical cyclones originate only over the seas and they dissipateon reaching the land. It is because they are supported by the moisture from the sea.So, 1 is correct.

On the equatorward side of the subtropical ridge, general easterly winds prevail.Also, there is an axis of high pressure called the subtropical ridge that extends east-west poleward of the storm. They steer the tropical cyclones to the west from theeast. So, 2 is correct.

Q Source:11th NCERT: Fundamentals of Physical Geography

76 India is the world's largest arms importer. Which two countries follow immediately inthe ranking of total arms imports?

China1.Australia2.Ukraine3.Myanmar4.Select the correct answer using the codes below.

1 and 2A.2 and 3B.1 and 4C.2 and 4D.

User Answer :Correct Answer : AAnswer Justification :

Justification:Top arms importers are: India (14%), China (4.7%), Australia (3.6%),Pakistan (3.3%), Vietnam (2.9%) and South Korea (2.6%).

India's arms imports remain three times greater than those of its rivalsPakistan and China. Its biggest suppliers are Russia, United States (US),Israel and France.This indicates India's failure to build a strong domestic defence-industrialbase (DIB) and Indian arms industry has failed to produce competitive

INSIG

HTS IAS

Page 51: Insight Test-13 Xaam.in

TEST - 13

User Name : chandan paswan

Total Marks : 200

Mark Scored : 22

(C) Insights Active Learning. | All rights reserved. www.insightsias.com 51

indigenously-designed weapons.Earlier China used to be top on the imports chart but in past few decades ithas gradually built a stronger DIB. China has emerged as the world's thirdlargest arms exporter after United States and Russia.

Q Source:Current Affairs

77 The passing of Election Laws (Amendment) Bill, 2016 in the Parliament wouldClarify provisions of the criminal justice system in case of convicted MPsA.and MLAsGive voting rights to people who became Indian citizens following theB.exchange of enclaves between India and BangladeshEnlarge the powers of Election Commission to investigate paid news and fakeC.accounts submitted by political partiesNone of the aboveD.

User Answer :Correct Answer : BAnswer Justification :

Learning: The historic land boundary agreement signed between India andBangladesh had altered the geography and demography of the district of CoochBehar in West Bengal.

Under this agreement, 51 Bangladeshi enclaves (Chhitmahals) were added in theIndian Territory and 111 Indian enclaves in Bangladesh territory.

Since the newly acquired area has become the part of Indian territory, it wasnecessary to make delimitation exercise within the limited constituency area ofCooch Behar before the ensuing 2016 West Bengal State Assembly elections.

Q Source:http://pib.nic.in/newsite/PrintRelease.aspx?relid=136631

78 The Union Cabinet has approved the LIGO-India project for research onGravitational wavesA.NeutrinosB.Higgs BosonC.Anti-matterD.

User Answer : ACorrect Answer : AAnswer Justification :

INSIG

HTS IAS

Page 52: Insight Test-13 Xaam.in

TEST - 13

User Name : chandan paswan

Total Marks 200

Mark Scored 22

(C) Insights Active Learning. | All rights reserved. www.insightsias.com 52

Learning:LIGO-India project is piloted by Department of Atomic Energy (DAE)and Department of Science and Technology (DST). It will give impetus to scientificresearch in the country.

It will establish a state-of-the-art Gravitational Wave Observatory in India incollaboration with the US based LIGO Laboratory run by Caltech and MIT.

It will also bring considerable opportunities in cutting edge technology in India'sscientific industry as domestic players will engaged in the construction of 8kilometre long beam tube at ultra-high vacuum of observatory on a levelled terrain.

Q Source:http://www.thehindu.com/sci-tech/science/indian-gravitational-wave-observatory-gets-union-cabinet-approval/article8248745.ece

79 Consider the following statements.Assertion (A): The system of monarchy cannot exist in a Parliamentary democracy.1.Reason (R): In a democracy representatives are elected based on popular vote.2.In the context of the above, which of these is correct?

A is correct, and R is an appropriate explanation of A.A.A is correct, but R is not an appropriate explanation of A.B.A is incorrect, but R is correct.C.Both A and R are incorrect.D.

User Answer : ACorrect Answer : CAnswer Justification :

Justification:Democracy is of two types - republic (where the head of state iselected, like India), and monarchy (where the head of state is hereditary).

UK practices Monarchy (crown) and yet it is a Parliamentary democracy.

Q Source:11th NCERT: Indian constitution at Work

80 Japan has recently launched ASTRO-H space observation satellite to probeExpansion of the Solar systemA.Mystery of the Black holesB.Generation and decay of space dustC.Possibility of finding water on PlutoD.

User Answer :

INSIG

HTS IAS

Page 53: Insight Test-13 Xaam.in

TEST - 13

User Name : chandan paswan

Total Marks : 200

Mark Scored : 22

(C) Insights Active Learning. | All rights reserved. www.insightsias.com 53

Correct Answer : BAnswer Justification :

Learning:It is a powerful orbiting observatory developed by the Japan AerospaceExploration Agency (JAXA) for studying extremely energetic processes in theuniverse.

It will orbit earth at an altitude of about 580 kilometres and observe X-raysemanating mainly from black holes and galaxy clusters.

Q Source: http://science.nasa.gov/missions/astro-h/

81 Consider the following about Wassenaar Arrangement.It concerns with transfers of conventional arms and dual-use goods and1.technologies.It is a sub-treaty under the Missile Technology Control Regime (MTCR).2.India is a member of the Wassenaar Arrangement.3.It does not extend to web-based systems.4.Select the correct answer using the codes below.

1, 2 and 4 onlyA.1 onlyB.2 and 3 onlyC.1, 2, 3 and 4D.

User Answer :Correct Answer : BAnswer Justification :

Justification:Statement 1: It promotes transparency and greater responsibility intransfers of conventional arms and dual-use goods and technologies, thus preventingdestabilising accumulations. The aim is also to prevent the acquisition of these itemsby terrorists.

Statement 2: Australia Group, Nuclear Suppliers Group and Missile TechnologyControl Regime are different arrangements than Wassenar.

Statement 3: India and China are non-participating states.

Statement 4: New technologies (amendment in 2013) placed under the exportcontrol regime include "intrusion software"-software designed to defeat a computeror network's protective measures so as to extract data or information-as well as IPnetwork surveillance systems.

INSIG

HTS IAS

Page 54: Insight Test-13 Xaam.in

TEST - 13

User Name : chandan paswan

Total Marks 200

Mark Scored 22

(C) Insights Active Learning. | All rights reserved. www.insightsias.com 54

Q Source: International treaties - Test 13 syllabus

82 The National Capital Goods Policy 2016 will focus onIncreasing imports of capital goods to satisfy domestic demand and revive economic1.growthMake standards mandatory in order to reduce sub-standard machine imports2.Opening Export processing zones (EPZs) in select countries to manufacture capital3.goods to reduce costSelect the correct answer using the codes below.

1 onlyA.2 and 3 onlyB.2 onlyC.1 and 3 onlyD.

User Answer :Correct Answer : CAnswer Justification :

Justification: Statement 1: It instead aims at increasing exports of capital goodsfrom the current 27% to 40 % of production, and increasing share of domesticcapital goods production in terms of demand from 60% to 80% in order to makeIndia a net exporter of capital goods.

Statement 2: It also seeks to make standards mandatory in order to reduce sub-standard machine imports and provide opportunity to local manufacturing units andlaunch scheme of skill development for Capital Goods sector.

Statement 3: There is no such proposal in the policy.

Q Source:http://pib.nic.in/newsite/PrintRelease.aspx?relid=136430

83 Which of the following organization(s) is/are specialized agencies of the United Nations(UN)?

World Trade Organization (WTO)1.Food and Agriculture Organization (FAO)2.World Health Organization (WHO)3.International Monetary Fund (IMF)4.Select the correct answer using the codes below.

2, 3 and 4 onlyA.1 and 4 onlyB.2 and 3 onlyC.

INSIG

HTS IAS

Page 55: Insight Test-13 Xaam.in

TEST - 13

User Name : chandan paswan

Total Marks : 200

Mark Scored : 22

(C) Insights Active Learning. | All rights reserved. www.insightsias.com 55

1, 2, 3 and 4D.User Answer : DCorrect Answer : AAnswer Justification :

Learning: WTO is a related organization (to the UN) founded in 1995 pertaining tothe GATT. It is not its specialized agency as 2, 3 and 4 are.

Other specialized agencies are IAEA, IMO, ILO, World Bank Group, WIPO etc.The complete list is here

https://en.wikipedia.org/wiki/List_of_United_Nations_organizations#Specialised_organisations_and_agencies

Q Source: Test-13 Syllabus - International Bodies

84 Consider the following statements:Indian economy nearly stagnated under colonial rule.1.Foreign investment was discouraged in the early decades post-independence.2.Central economic planning in India started only after independence.3.Which of the above is/are correct?

2 onlyA.1 and 2 onlyB.3 onlyC.1, 2 and 3D.

User Answer : DCorrect Answer : BAnswer Justification :

Justification: Central economic planning started with the setting up of the NationalPlanning Committee (1938). So, 3 is wrong.

Foreign investment was discouraged because of the colonial experience of Drain ofwealth and the stagnant performance of the economy under the colonial rule. It grewby zero % between 1600 and 1870, and a meagre 0.2 % between 1870 to 1947.

So, 1 and 2 are correct.

Q Source:11th NCERT: Indian Economic Development

INSIG

HTS IAS

Page 56: Insight Test-13 Xaam.in

TEST - 13

User Name : chandan paswan

Total Marks 200

Mark Scored 22

(C) Insights Active Learning. | All rights reserved. www.insightsias.com 56

85 Consider the following statements:Crude Oil and Cement related industries are a part of 'core industries' in India.1.Steel has the highest weightage in the Index of Industrial production (IIP) amongst2.the 'core industries'.Which of the above is/are true?

1 onlyA.2 onlyB.Both 1 and 2C.NoneD.

User Answer :Correct Answer : AAnswer Justification :

Justification: Eight core industries are - Iron and Steel, Cement, Crude Oil, NaturalGas, Fertilizers, Oil refining, Electricity and Coal. They together have a weight of37.90% in the IIP.

Out of these electricity has the highest weightage amongst them of nearly 10.32%.Steel has the second highest weightage of nearly 6.68 %.

Q Source: 11th NCERT: Indian Economic Development and Past year UPSC CSPpapers

86 Consider the following statements about the source of capital for the states in India.Finance commission does not generally determine capital related issues of the state1.and is confined to revenue matters and grants-in-aid only.The FRBM Act has restricted the freedom of the states to borrow for fulfilling their2.plan expenditure.Which of the above is/are true?

1 OnlyA.2 OnlyB.Both 1 and 2C.NoneD.

User Answer :Correct Answer : AAnswer Justification :

Justification: Statement 1: Although the constitution does not define revenue andcapita related matters, the job of finance commission is restricted. The capitalrelated issues of the states are taken care of by the Planning Commission which is

INSIG

HTS IAS

Page 57: Insight Test-13 Xaam.in

TEST - 13

User Name : chandan paswan

Total Marks : 200

Mark Scored : 22

(C) Insights Active Learning. | All rights reserved. www.insightsias.com 57

seen to eclipse the role of a constitutional body FC.

Statement 2: The FRBM Act does restrict the fiscal deficit of states by giving them adirection. But, it gives them further freedom to borrow from the markets in case theyhave enacted their own fiscal responsibility legislations. They can thus fulfill theirplan expenditure. So, 2 is wrong.

Q Source: 12th NCERT: Macroeconomics

87 What was/were the change(s) introduced in the Indian governance by the Governmentof India Act 1858?

The act abolished the rule of East Indian Company, and established that of British1.crown.All civil, military and executive powers were to be vested in the bureaucracy.2.It increased the legislative autonomy of the Princely states and British provinces.3.Select the correct answer using the codes below.

1 and 3 onlyA.1 onlyB.1 and 2 onlyC.2 and 3 onlyD.

User Answer :Correct Answer : BAnswer Justification :

Justification:The Company's territories in India were to be vested in the Queen, theCompany ceasing to exercise its power and control over these territories. India wasto be governed in the Queen's name. So, 1 is correct.

The administration of the country was now highly centralized. All civil, military andexecutive powers vested in the Governor in council, who in turn was responsible toSecretary of State. So, both 2 and 3 are incorrect.

Learning:There was also a provision of creation of an Indian Civil Service underthe control of the Secretary of State.

This act abolished the Dual Government (Board of Control and Board ofDirectors in the company) introduced by the Pitt's India act.

Q Source:8th NCERT: History

INSIG

HTS IAS

Page 58: Insight Test-13 Xaam.in

TEST - 13

User Name : chandan paswan

Total Marks 200

Mark Scored 22

(C) Insights Active Learning. | All rights reserved. www.insightsias.com 58

88 Agricultural produce market committees (APMC) are established by theAuthorized Local area Vendors (ALAVs)A.Farmer Produce Organizations (FPOs)B.State GovernmentC.Local GovernmentD.

User Answer : CCorrect Answer : CAnswer Justification :

Learning:APMC operate on two principles:

Ensure that farmers are not exploited by intermediaries (and money lenders)who compel farmers to sell their produce at the farm gate for an extremelylow price.All food produce should first be brought to a market yard and then soldthrough auction.

Each state which operates APMC markets geographically divide the state andmarkets (mandis) are established at different places within the state.

Farmers are required to sell their produce via auction at the mandi in their region.Traders require a license to operate within a mandi.

Wholesale and retail traders (e.g. shopping mall owners) and food processingcompanies cannot buy produce directly from a farmer.

Q Source:Past year UPSC CSP papers

89 In which of the following cases of constitutional amendment, the Supreme Court canpossibly declare the bill as null and void for violating the basic structure of theconstitution?

An amendment which declares the Parliament to be supreme and above the1.Judiciary.An amendment which makes India a police state instead of a welfare state.2.An amendment which gives the Parliament substantial judicial powers.3.Select the correct answer using the codes below.

1 and 2 onlyA.1 and 3 onlyB.2 and 3 onlyC.1, 2 and 3D.

User Answer : D

INSIG

HTS IAS

Page 59: Insight Test-13 Xaam.in

TEST - 13

User Name : chandan paswan

Total Marks : 200

Mark Scored : 22

(C) Insights Active Learning. | All rights reserved. www.insightsias.com 59

Correct Answer : DAnswer Justification :

Justification:The basic structure of the constitution contains these, amongst others:Welfare state; Separation of powers; Supremacy of the constitution.

All these three cases try to disturb the basic structure. Thus they can be termed voidby the Supreme Court.

Q Source: 11th NCERT: Indian constitution at Work

90 Which of the following CANNOT be the reason(s) for the downfall of the HinduEmpires in India to the incoming Muslim rulers?

The Hindu states were tied under a common federation thus devoid of exercising1.power.Their military methods were out of date and inferior to those of Muslims.2.Which of the above is/are correct?

1 onlyA.2 onlyB.Both 1 and 2C.NoneD.

User Answer : BCorrect Answer : AAnswer Justification :

Justification & Learning: The most important cause for the downfall of Hindustates was that they lacked unity. There was no formal coordination mechanism.They were divided by factions. For e.g. the Rajput princes exhausted one another bytheir mutual conflicts. So, 1 cannot be the reason.

Also, many Hindu states were declining in power. Their military methods were outof date and far inferior to those of Muslims. Indians continued to rely on elephantswhile the Muslims possessed quick-moving cavalry. So, 2 can be a reason.

Other reasons can be:

The Muslims soldiers had better organization and able leaders.Their religious zeal and their greed for the greater wealth of India providedstimulus to them.Among the Hindus, the duty of fighting was confined to a particular class, theKshatriyas.

INSIG

HTS IAS

Page 60: Insight Test-13 Xaam.in

TEST - 13

User Name : chandan paswan

Total Marks 200

Mark Scored 22

(C) Insights Active Learning. | All rights reserved. www.insightsias.com 60

Moreover, the Hindus were always on the defensive, which was considered tobe a weak position in war affairs then.

Q Source: 11th TamilNadu History Textbook

91 Consider the following about the United Nations Security Council (UNSC).It is the only UN body with the authority to issue binding resolutions to member1.states.All decisions of the UNSC must be approved by the UN General Assembly to be2.enforced.It recommends new states for admission as member states of the United Nations3.(UN).Which of the above is/are correct?

1 and 2 onlyA.2 onlyB.1 and 3 onlyC.None of the aboveD.

User Answer : DCorrect Answer : CAnswer Justification :

Justification: Decisions that are vetoed by the P-5 cannot be submitted for thereview of the UNGA. Also, the UNSC is the authority under the UN charter to takeaction in situations

threatening international peace and security, e.g. Syria crisis. So, 2 is incorrect.

Decisions taken under Chapter VII, such as economic sanctions or other sectionslike international security, are binding on UN members; the Security Council is theonly UN body with the authority to issue binding resolutions. The UNGA can onlymake recommendations in this regard. So, 1 is correct.

Q Source: Test-13 Syllabus - International Bodies

92 The Earth's geomagnetic field is a combination of several magnetic fields generated byvarious sources which include?

Movement of conducting material inside the Earth's core1.Electric current flowing in the ionized upper atmosphere2.Currents flowing within the earth's crust3.Select the correct answer using the codes below.

INSIG

HTS IAS

Page 61: Insight Test-13 Xaam.in

TEST - 13

User Name : chandan paswan

Total Marks : 200

Mark Scored : 22

(C) Insights Active Learning. | All rights reserved. www.insightsias.com 61

1 and 2 onlyA.1 onlyB.2 and 3C.1, 2 and 3D.

User Answer :Correct Answer : DAnswer Justification :

Learning: More than 90% of the field is generated by the movement of conductingmaterial inside the Earth's core, which is often referred to as the Main Field. Otherimportant sources of the geomagnetic field include electric current flowing in theionized upper atmosphere and currents flowing within the earth's crust.

There are also local anomalies produced by mountain ranges, ore deposits,geological faults, and artificial products such as trains, aircraft, power lines etc.

Q Source: Geomagnetism topic in 11th NCERT: Fundamentals of PhysicalGeography

93 Consider the following statements about the Securities and Exchange Board of India(SEBI).

It derives its authority solely from executive orders.1.It registers stock exchanges and mutual funds.2.It promotes financial literacy in the country.3.It is tasked with the inspection and audit of stock exchanges.4.Select the correct answer using the codes below.

2, 3 and 4 onlyA.1 and 3 onlyB.2 and 4 onlyC.1, 2, 3 and 4 onlyD.

User Answer : CCorrect Answer : AAnswer Justification :

Justification: It was initially a non-statutory body setup by a government order. Itwas afterwards given powers under the SEBI Act, 1992. So, 1 is wrong.

It is the principal agency in India which regulates the security market and promotesinvestor education.

This link (though not authoritative) explains the functions of SEBI in detail.

INSIG

HTS IAS

Page 62: Insight Test-13 Xaam.in

TEST - 13

User Name : chandan paswan

Total Marks 200

Mark Scored 22

(C) Insights Active Learning. | All rights reserved. www.insightsias.com 62

http://www.yourarticlelibrary.com/education/sebi-the-purpose-objective-and-functions-of-sebi/8762/

Q Source:Past year UPSC CSP papers

94 Which of the following form part of the revenue expenditure of the Government ofIndia?

Expenditure on social security1.Grants given to foreign countries and Indian states2.Subsidies forwarded to all sectors of the economy3.Select the correct answer using the codes below.

1 and 2 onlyA.2 and 3 onlyB.1 and 3 onlyC.1, 2 and 3D.

User Answer : DCorrect Answer : DAnswer Justification :

Justification: Loans given to Indian states qualify as capital expenditure. But grantsto either an Indian or a foreign state qualify as revenue expenditure.

Generally, expenditure incurred on normal running of the government departmentsand maintenance of services is treated as revenue expenditure.

Examples of revenue expenditure are salaries of government employees, interestpayment on loans taken by the government, pensions, subsidies, grants, ruraldevelopment, education and health services, etc.

Q Source: 12th NCERT: Macroeconomics

95 Which of the following CANNOT be the cause(s) for the Decline of Buddhism in India?Revival of Brahmanism and the rise of Bhagavatism1.Use of Sanskrit instead of common man's language Pali2.Destruction of monasteries by Turkish invaders3.Select the correct answer using the codes below.

1 onlyA.2 onlyB.2 and 3 onlyC.None of (a), (b) or (c) is correct.D.

INSIG

HTS IAS

Page 63: Insight Test-13 Xaam.in

TEST - 13

User Name : chandan paswan

Total Marks : 200

Mark Scored : 22

(C) Insights Active Learning. | All rights reserved. www.insightsias.com 63

User Answer : BCorrect Answer : DAnswer Justification :

Justification: Statement 1: The revival of Brahmanism and the rise of Bhagavatismled to the fall of popularity of Buddhism.

Statement 2: The use of Pali, the language of the masses as the language ofBuddhism was given up from the 1st century A.D. The Buddhists began to adoptSanskrit, the language of the elite.

Also, after the birth of Mahayana Buddhism, the practice of idol worship andmaking offerings led to the deterioration of moral standards.

Statement 3: Moreover, the attack of the Huns in 5th and 6th centuries and theTurkish invaders in 12th century destroyed the monasteries.

Q Source: 11th Tamilnadu History Textbook

96 Which of the following are responsible for bringing changes on the surface of the earth?Gravitational force1.Tectonic forces2.Electromagnetic radiation3.Select the correct answer using the codes below.

1 and 2 onlyA.2 and 3 onlyB.1 and 3 onlyC.1, 2 and 3D.

User Answer : ACorrect Answer : DAnswer Justification :

Justification:Statement 1: Gravity causes movement of ocean current, induceslandslides, influences river movements etc causing dynamic change on landscape ofearth.

Statement 2: Tectonic forces can build mountains as well as large rifts.

Statement 3: Sun's radiation, i.e. light, is electromagnetic in nature. All landformand life on earth ultimately comes from the Sun.

INSIG

HTS IAS

Page 64: Insight Test-13 Xaam.in

TEST - 13

User Name : chandan paswan

Total Marks 200

Mark Scored 22

(C) Insights Active Learning. | All rights reserved. www.insightsias.com 64

Q Source:11th NCERT: Fundamentals of Physical Geography

97 CITES (the Convention on International Trade in Endangered Species of Wild Faunaand Flora) is an international agreement between

Governmens1.GovernmentsA.NGOsB.Conservation CentresC.Regional Environment BodiesD.

User Answer :Correct Answer : AAnswer Justification :

Learning: It aims to ensure that international trade in specimens of wild animalsand plants does not threaten their survival. Roughly 5,000 species of animals and28,000 species of plants are protected.

Bears, dolphins, cacti, corals, orchids and aloes are some examples.

Q Source: 9th NCERT: Geography

98 Coalbed Methane in India can be found inJharkhand1.Madhya Pradesh2.West Bengal3.Select the correct answer using the codes below.

1 and 2 onlyA.1 onlyB.2 and 3 onlyC.1, 2 and 3D.

User Answer :Correct Answer : DAnswer Justification :

Learning: India has the fifth-largest coal reserves in the world, estimated at morethan 300 billion tonnes by the Ministry of Coal as in 2014.

This means significant potential for CBM exploitation.

The exploration of CBM in India is spread over a sedimentary area of 26,000 square

INSIG

HTS IAS

Page 65: Insight Test-13 Xaam.in

TEST - 13

User Name : chandan paswan

Total Marks : 200

Mark Scored : 22

(C) Insights Active Learning. | All rights reserved. www.insightsias.com 65

kilometres and is already ongoing, mainly regions such as Jharkhand, West Bengaland Madhya Pradesh.

Q Source:http://envfor.nic.in/sites/default/files/cc/cop8/moefbk/use.pdf

99 Consider the following about the Intergovernmental Panel on Climate Change (IPCC).It is a scientific intergovernmental group established by the UNFCCC.1.Membership of the IPCC is open to all member countries of the United Nations2.(UN).It conducts authoritative research on climate related issues.3.It monitors climate related parameters such as Mean Sea Level (MSL).4.Its recommendations are binding on all members of the UNEP.5.Select the correct answer using the codes below.

1, 4 and 5 onlyA.1, 2, 3 and 4 onlyB.2 onlyC.1, 2, 3, 4 and 5D.

User Answer :Correct Answer : CAnswer Justification :

Justification: IPCC is the leading international body for the assessment of climatechange.

It was established by the United Nations Environment Programme (UNEP) and theWorld Meteorological Organization (WMO) in 1988 to provide the world with aclear scientific view on the current state of knowledge in climate change and itspotential environmental and socio-economic impacts. So, 1 is wrong.

The IPCC reviews and assesses the most recent scientific, technical and socio-economic information produced worldwide relevant to the understanding of climatechange. It does not conduct any research nor does it monitor climate related data orparameters. So, 3 and 4 are wrong.

As an intergovernmental body, membership of the IPCC is open to all membercountries of the United Nations (UN) and WMO. Currently 195 countries areMembers of the IPCC. So, 2 is correct.

QSource:http://www.moef.gov.in/sites/default/files/treaties/international-treaties.html

INSIG

HTS IAS

Page 66: Insight Test-13 Xaam.in

TEST - 13

User Name : chandan paswan

Total Marks 200

Mark Scored 22

(C) Insights Active Learning. | All rights reserved. www.insightsias.com 66

100 Stockholm Convention on Persistent Organic Pollutants (POPs) aims to eliminate orrestrict the production and use of POPs. Which of the following is/are POPs listed underthe convention?

Heptachlor1.DDT2.Endosulfan3.Mercury4.Lead5.Select the correct answer using the codes below.

1, 4 and 5 onlyA.2, 3, 4 and 5 onlyB.2 and 3 onlyC.1, 2 and 3 onlyD.

User Answer :Correct Answer : DAnswer Justification :

Learning: Heptachlor is used as a termiticide (including in the structure of housesand underground), for organic treatment and in underground cable boxes.

Endosulfan is an off-patent organochlorine insecticide and acaricide that is beingphased out globally.

DDT and other pesticides had been shown to cause cancer and that their agriculturaluse was a threat to wildlife, particularly birds.

Unlike the persistent organic pollutants (POPs), metals Mercury do not accumulatein fat. Hence, they are not POPs.

QSource:http://www.moef.gov.in/sites/default/files/treaties/international-treaties.html

INSIG

HTS IAS